Вы находитесь на странице: 1из 48

L

U
o
z
,-
VI
01 .
1.l.
1.2.
02.
2.l.
2.2.
2.3.
2.4.
2.5.
Semiologa urolgica
y definiciones 01
Defniciones 01
Diagnstico diferencial
de la hematuria macroscpica 02
Infecciones
del tracto urinario.
Cistitis intersticial 03
Patognesis y etiologa 03
Diagnstico 04
Diferentes ITU y su tratamiento 04
Tuberculosis genitourinaria 07
Cistitis intersticial 07
03. Urolitiasis 09
3.l. Epidemiologia 09
3.2. Manifestaciones clnicas
y su manejo agudo 10
3.3. Evaluacin y tratamiento
de la litiasis renal 11
04. Tumores renales 15
4.1. Carcinoma de clulas renales
(adenocarcinoma renal, hipernefroma) 15
4.2. Otros tumores 17
05. Hiperplasia
y carcinoma prosttico 19
S.l. Hiperplasia prosttica benigna 19
5.2. Carcinoma prosttico 21
06. Carcinomas 09. Uropata obstructiva 38
del tracto urinario 26
9.l. Caractersticas 38
9.2. Clnica 38
6.l. Carcinoma vesical 26
9.3. Diagnstico 39
6.2. Tumores del tracto urinario superior 28
9.4. Tratamiento 39
07. Tumores testiculares 31
1 0. Disfuncin erctil 40
7.1. Etiologa y epidemiologa 31
10.1. Introduccin 40
7.2. Anatoma patolgica 32
10.2. Prevalencia 40
7.3. Clnica 32
10.3. Etiologa 40
7.4. Diagnstico 32
10.4. Factores de riesgo 40
7.5- Diagnstico diferencial 33
10.5- Diagnstico 41
7.6. Tratamiento 34
10.6. Tratamiento 41
08. Trasplante renal 36
Bibl iografa 42
8.l. Indicaciones 36
8.2. Contraindicaciones 36
8.3. Complicaciones 37
VII
01
SEMIOLOGA UROLGICA
` DEFINICIONES
Orientacin
MIR
Este tema no s ha preguntado
en el MIR de forma directa.
Puede ayudar a obtener una
visin general de la materia
ya asociar algunos hallazgos
a patologras concreas, pero
no se debe emplear en l
demasiado tiempo.
? Preguntas
- No hay preuntas MIR
representativas.
Aspectos esenciales
G
II
La causa ms frecuente de hematuria microscpica es la litiasis (poblacin general, ambos sexos).
la causa ms comn de hematuria microscpica en varones de ms de 50 aos es la hiperplasia benigna
de prstata.
la hematuria con cogulos indica un problema urolgico.
la causa ms habitual de hematuria es la cistitis hemorrgica, pero 10 primero a descartar es el tumor uro
leJia!.
I
Los hemates dismrficos en el sedimento orientan a nefropala de origen glomerular.
1 . 1 . Defni ci ones
Hematuria microscpica: presencia de ms de 5 hemates por campo. La causa ms frecuente en ambos sexos
es la l i tiasis. La causa mas comn en varones mayores de 50 aos es la hiperplasia benigna de prstata.
Hematuria macroscpica: orina de aspecto rojizo a simple vista debido a la presenci a de ms de 50 hemates
por campo. En los pacientes fumadores, en ausencia de otros sntomas, se debe sospechar tumor urotel i al .
Piuria: presencia de ms de 10 leucocitos por campo. Altamente nespecfica, pero en presenci a de sntomas
uri narios, hay que sospechar infeccin.
Sndrome miccional: presencia de polaquiuria (aumento en la frecuencia miccional), urgencia miccional (ne
cesidad imperiosa e irrefrenable de orinar) y disuria (molestias urinarias inespecficas referidas como ardor,
escozor, etc.).
Incontinencia urinaria: prdidas involuntarias de orina (Tabla 1 l. Existen cuatro ti pos principales:
SINTOMAS
INCONTINENCIA INCONTINENCIA
DE URGENCIA DE ESFUERZO
Ur (d rno de ortnar) Si No
Aumento de laheuencla ml Si No
Cldod de _ .. bolo despus de _ 01 deseo de orinar No SI
_Ir"boIo_la_ sr Generalmente no
Escape _la l f No Si
eanddod de orIno .. poda en coda opIldlo de In"''cla Abundante. si se produce Generalmente escasa
Tabla 1. Diagnstico diferencial de la cl nica de incontinencia urinaria femenina
Continua: de da y de noche, en todas las posiciones. La causa ms frecuente es la fstula urinaria (en pacientes
con antecedentes quirrgicos previos) y la segunda, el urter ectpico (que es la causa ms frecuente en ni as).
De esfuerzo: se desencadena con el aumento de presin abdomi nal (al rer, toser, cargar con peso). Ge
neralmente se produce por un dficit de soporte de la musculatura perineal (por el l o es recomendable
revisar los antecedentes obsttricos, pacientes obesas, pacientes aosas, ete.).
De urgencia: el paciente siente ganas de orinar, pero no le da tiempo a l legar al bao (provocadas por
contracciones involuntarias del msculo detrusor).
Manual CTO de Medicina y Cirugfa, 8.a edicin
Mixta: generalmente es una combinacin de las dos anteriores.
Paradjica: escape de orina debido a la sobreclistensin vesical.
El ejemplo caracterstico es el paciente prosttico con retencin
urinaria. La presin i ntravesical supera la presin de cierre del
esfnter uretral, producindose un escape de orina paradj ico
(no puede orinar y, sin embargo, se l e escapa l a orina).
RECUERDA
2
los cilindros hemticos aparecen en las glomerulonefritis que producen
sndrome nefrtico, como en la postinfecciosa.
Enuresis: prdidas de orina exclusivamente durante el sueo. Si el
nio es mayor de 6 aos, debe ser estudiado.
Crisis renoureteral: dolor lumbar frecuentemente irradiado a geni
tales, de carcter agudo, cuya i ntensidad no se modifica por los
cambios pastura les, y que se suele acompaar de nuseas, vmitos
y malestar general. Es muy poco frecuente que sea bi lateral.
1 .2. Di agnstico di ferenci al
de l a hematuri a macroscpica
Segn el momento de aparicin:
Inici al: sangrado uretral o prosttico.
Fi nal: sangrado del cuello vesical.
Total: vesical o del tracto urinario alto. Se debe recordar que un
sangrado importante de cualquier parte del aparato genitourina
rio puede provocar hematuria total.
Hematuria con cogulos: indica un problema urolgico. La cau
sa ms frecuente en mujeres es la cistitis hemorrgica, aunque l a
primera causa a descartar es una neoplasia urotelial, mxime en el
paciente fumador.
Hematuria por nefropata mdica: no suele tener cogulos, y puede
ir acompaada de cierto grado de protei nuria, as como de ci l i ndros
eritrocitarios o de hemates dismrficos en el sedimento urinario.
02.
INFECCIONES DEL TRACTO URINARIO.
CISTITIS INTERSTICIAL
Orientacin
MIR
Este tema es el ms importante
deesta asignatura. Se debe
conocer muy bien, pues dos
o tres preguntas son habituale
en el examen. El estudio de las
preguntas de otros aos suele
ser de gran ayuda, ya que son
baStlnle repetitivas. No se
debe bajar la guardia con l a
tuberculosis genitaurinaria ni
en la cistitis intersticial. Hay
que formar una imagen mental
tpica para reconocerlas en
caso clnico, que es como
suelen preguntarlas.
II Preguntas
-MIR 08-09, 98
- MIR 05-06, 230
-MIR 03-04, 78,84
-MIR 02-03, 1 34, 1 74
-MIR OJ-02, 103, 1 06
-MIR OO-01F, 1 34, 1 45
- MIR 99-0, 1 35
- MIR 99-00f,1 1 9
- MIR 98-99F, 1 1 8
- MIR 97-98, 26, 206, 21 5
o
la causa ms frecuente de infeccin del tracto urinario OTU) es Escherichia coli, tanto a nivel comunitario
como nosocomial.
El origen ms frecuente de uretritis es Chlamydia lrachomalis.
la causa habitual de orquiepididimitis depende de la edad: Chlamydia y gonococo si es menor de 35 aos;
enterobacterias, si es mayor de esa edad.
la causa ms frecuente de absceso renal en el UDVP es Staphylococcus aureus.
El diagnstico definitivo de ITU es microbiolgico: ms de '05 UFClm1. No obstante, este criterio vara con
el sistema de recogida.
Si se recoge la muestra urinaria mediante puncin suprapbica, cualquier nmero de bacterias e significativo.
la bacteriuria asintomtica se trata en gestantes, menores de 5 aos, inmunodeprimidos, previamente a la
ciruga urolgica, o si la especi e implicada es Prteus.

Los sistemas de drenaje cerrados son preferibles a los abiertos, pues la lasa de infeccin es menor.
La infeccin del tracto urinario (lTU) puede clasificarse de varias formas. Se puede hacer una divisin anatmica
entre las ITU altas (i nfecciones renales) y las ITU bajas (cistouretritis, prostatitis). Asimismo, la clasificacin pue
de basarse en la asociacin o no de complicaciones. Una ITU no complicada es un cuadro clnico caracterizado
por l a presencia de escozor miccional, urgencia y frecuencia, acompaado o no por hematuria terminal, dolor
hipogstrico, y ms raramente, febrcula. Dentro de este grupo se podran i ncl ui r las pielonefritis no complica
das, que se presentan como cuadros febriles con hipersensibilidad en fosa lumbar, fiebre, nuseas o vmitos, y
sin los factores que convierten la [TU en "complicada", como son: presencia de catteres, uropata obstructiva,
reflujo vesicoureteral, anomalas anatmicas, insuficiencia renal o trasplante renal. La ITU en el varn debe
considerarse esencialmente "complicada" de entrada.
La reaparicin de una infeccin tras el tratamiento puede deberse a reinfeccin o recidiva. El primer trmino expre
sa la infeccin nueva por un germen distinto al inicial, mi entras que recidiva indica infeccin por el mismo germen.
Esta ltima es mucho ms infrecuente que la reinfeccin y puede estar ocasionada por litiasis infectiva, prostatitis
crnica, fstulas vaginales o i ntestinales, divertculos vesicales i nfectados, cuerpos extraos, necrosis papilar i nfec
tada y otras causas que generan un reservario de microorganismos que difcilmente se el imi nan con el antibitico.
2. 1 . Patognesi s yeti ol oga
Existen tres posibles vas por las que los microorganismos pueden alcanzar el tracto urinario: hematgena,
l i nftica y ascendente. La va l inftica carece de i mportancia real. La diseminacin hematgena tampoco es
ferecuente. La ms comn es la ascendente i niciada en la uretra. Probablemente por esta razn es mucho ms
habitual l a ITU en mujeres, dado que su uretra es muy corta y ancha, y por ello favorece el paso de microorga
nismos hacia niveles ms altos del TGU.
Otro dato que apoya l a importancia de l a va ascendente es l a frecuencia de i nfeccin tras el cateterismo
uretral, que es del 1 % en los pacientes ambulantes, y en tres o cuatro das alcanza a casi la total i dad de los
pacientes sondados con sistemas de drenaje abiertos. En los enfermos hospitalizados, el riesgo de i nfeccin
alcanza un 5% por cada da de sondaje, incluso con sistemas cerrados (MI R 01 -02, 103).
3
Manual CTO de Medicina y Ci ruga, 8.a edicin
Una vez que las bacterias han alcanzado el tracto urinario, tres factores
determi nan el desarrollo de la infeccin:
La virulencia del microorganismo.
El tamao del inculo.
Los mecanismos de defensa del husped.
La mayora de las infecciones en l a comunidad estn producidas por
grmenes gramnegativos, principalmente E. coli (MI R 05-06, 230), res
ponsable del 85% y, en menor proporcin, Proteus, Klebsie/la o Pseu
domonas (MIR 97-98, 21 5). Entre los grampositivos, nicamente el Sta
phylococcus saprophyticus tiene relevancia, produciendo el 10- 1 5%
de las ITU en mujeres jvenes (segundo germen ms frecuente en esta
poblacin).
Alrededor del 30% de las mujeres con clnica miccional presentan re
cuentos menores de 105 colonias por mi l i l i tro ( 1 05 UFCml); de stas,
tres cuartas partes presentan piuria; en el resto, existen pocos datos
que demuestren i nfeccin, y en general se tratan segn la clnica. En
la orina de las paci entes si ntomticas con piuria, se pueden encontrar
(considerndose infeccin activa) recuentos ms bajos (1 02- 1 04) de
los patgenos habituales. En otras ocasiones, el cuadro se justifica por
la presencia de uretritis causada por N. gonorrhoeae o C. trachomatis.
El papel patgeno de grmenes como U. urealyticum o Mycoplasma
hominis est mal definido, ya que se desconoce su potenci al como
uropatgenos aislados (MIR 99-00F, 119).
En las i nfecciones nosocomiales, los grmenes gramnegativos conti
nan siendo los ms frecuentes. Si bien E. coli es el ms habitual, su
frecuencia desciende hasta el 50% y adquieren mayor importancia
Proteus, Klebsiella, Pseudomonas, Enlerobacler y Serratia (MI R 03-04,
84). El 25% restante est ocasionado por grmenes grampositivos como
estreptococos y estafilococos. Candida albicans puede aparecer prin
cipalmente en pacientes diabticos, cateterizados o con tratamientos
antibiticos prolongados.
la afectacin del tracto urinario superior parece tambin producirse
por ascenso de los grmenes a lo largo del urter. La diferenciacin,
aunque poco especfica, se debe basar en los hal lazgos clnicos (fiebre,
dolor l umbar, escalofros) y analtica elemental ( leucocitosis, velocidad
de sedimentacin alta).
2.2. Di agnstico
El di agnstico de ITU, adems de l a cl ni ca, se define por el cultivo
de ori na. Dado que es frecuente el crecimiento de bacterias que han
contaminado l as muestras, se uti l i za un criterio estadstico sobre l a
base del recuento de col oni as del urocultivo, considerando como
si gni ficativo clsicamente el crecimi ento de ms de 1 05 colonias por
mi l i l itro ( MI R 97-98, 206). En determinadas circunstancias, recuen
tos de colonias menores pueden ser suficientes: recuentos de 1 03
UFC/ml en mujeres si ntomticas, ms de 1 04 en pielonefritis cl nicas
o en varones, y ms de 1 02 en muestras de cateterismos l i mpios
o cual qui er recuento, si se recoge mediante punci n-
aspiracin su
prapbica (MIR 97-98, 26). Cifras mayores de 105 UFC/ml pueden
igualmente reflejar contami naci n, principalmente si crecen dos o
ms especies.
En el adulto, l a presencia de piuria (ms de 1 0 leucocitos/mm3) se rela
ciona estrechamente con la ITU en presencia de sntomas, no as en el
nio, en el que puede acompaar a los cuadros febriles.
4
2. 3. Di ferentes ITU y su tratami ento
En el tratamiento de la ITU lgicamente es fundamental el empleo de
antimicrobianos. El nmero de stos empleado es el evado y las pautas
de tratamiento muy variables. A conti nuacin, se repasarn las opcio
nes teraputicas segn el tipo de ITU a la que uno se enfrente.
Bacteri uri a asi ntomtica
Definida como bacteriuria significativa (105 UFC/ml) en al menos dos
urocultivos con el mismo germen, tomados con una semana de dife
rencia en ausencia de sntomas. la bacteri uri a asintomtica no debe
tratarse salvo en los casos en los que conlleva un ri esgo de i nfeccin
clnica o dao orgnico, como ocurre en nios menores de 5 aos,
tengan o no patologa urolgica asociada. Asimismo, debe ser tratada
en el embarazo (MIR 08-09, 98 ; MI R 03-04, 78), en pacientes inmu
nodepri mi dos, como profi laxis previa a una ciruga urolgica y en los
casos de bacteriuria por Proteus (MIR 02-03, 1 34) (Tabla 2).
Embarazadas
Inmunodeprimidos
Previamente a ciruga urolgica
Bacteriuria por Prteus
Tabla 2. Bacterruria asintomtica: indicaciones de tratamiento
RECUERDA
Prteus es intrnsecamente resistente t las nitrofurantonas, ya que al
caliniza la orina gracias a su ureasa, y este grupo de antibiticos nica
mente es til en medio cido.
En el caso de los pacientes sondados permanentemente, la presencia
de bacteri uria asi ntomtica no es una indicacin de tratamiento y,
actualmente, incluso es dudosa la recomendacin clsica de empleo
profilctico de algn anti bitico, previo a la sustitucin del catter, a
fin de contrarrestar la posible diseminacin hematgena del germen
producida por la manipulacin (las ltimas guas clnicas ya no lo reco
miendan). S es, sin embargo, i ndicacin de tratamiento la bacteriuria
persistente a los 3-5 das de haber retirado una sonda vesical. En aque
llos pacientes en los que l a sonda no pueda ser retirada, el tratamien
to de las bacteriurias asi ntomticas no suele ser efectivo, y puede dar
lugar a seleccin de cepas resistentes. En estos pacientes slo se debe
i ni ci ar tratamiento si presentan alto ri esgo de desarrollar bacteriemia o
si la bacteri uria se hace sintomtica.
En el resto de los casos, nicamente con la concurrencia de factores
particul ares, se debe tratar la bacteriuria, y siempre sobre la base del
estudio de sensibil idades (MI R 98-99F, 1 1 8) .
ITU baja en mujeres
Puede realizarse un tratamiento convencional de siete das o bien un
curso corto en monodosis o en rgimen de tres das. La ventaja de stos
es el menor coste econmico y la menor incidencia de efectos adver
sos. Su desventaja es la mayor incidencia de recurrencias tempranas, al
no afectar apenas a los reservorios vaginal e i ntestinal de uropatgenos.
Aun con todo, por las ventajas mencionadas, la pauta preferida actual
mente es el tratamiento de tres das.
Los antibiticos de eleccin son el cotrimoxazol, las fluoroquinolonas
y la amoxicili na-cido cl avul ni co, fosfomicina o nitrofurantona.
En mujeres embarazadas se recomiendan las pautas l argas de trata
miento (siete das), evitando el uso de sulfamidas al final del embarazo
por el riesgo incrementado de kernicterus, y el empleo de quinolonas
por el dao producido sobre el cartlago de crecimiento fetal . Tampoco
se emplearn pautas cortas en caso de sospecha de pielonefritis, pre
sencia de clculos o anomalas de la va urinaria, o bien i nfecciones
previas por microorganismos resistentes a los antibiticos.
ITU recurrente
Aparicin de cuatro o ms episodios al ao. Se puede realizar profi
laxis con cotrimoxazol O una fluoroquinolona (en funcin de la sensi
bilidad del germen aislado en el ltimo episodio) en dosis nica, das
alternos, durante seis meses. Si tras l a retirada se presentaran nuevas
recurrencias, puede reinstaurarse el tratamiento durante periodos ms
prolongados (1-2 aos). Es aconsejable la ingesta abundante de agua
y realizar micciones frecuentes y cumplir una serie de reglas bsicas
higinico-dietticas.
Si los episodios tienen relacin con el coito, se puede administrar un
comprimido de cotrimoxazol o una quinolona despus del mismo. En
mujeres posmenopusicas, el tratamiento con estrgenos tpicos vagi
nales disminuye l a frecuencia de i nfecciones.
RECUERDA
Staphylococcus saprophylicus se ha relacionado con ITU en mujeres
jvenes sexualmente activas.
Pielone!ritis aguda no complicada
En los casos de gravedad leve-moderada, puede plantearse terapu
tica oral con cotrimoxazol (en desuso en nuestro medio por el ele
vado ndice de resistencias), fluoroqui nolonas o B-
Iactmicos. En
pacientes graves u hospital i zados es preciso tratamiento parenteral,
y el espectro de anti microbianos i ncluye ampi ci l i na (enterococo),
ureidopeni ci l i nas (Pseudomonas), cefalosporinas de segunda o ter
cera generacin, e incluso aminoglucsidos. Nunca se emplearn
pautas cortas.
urologa _
El anti bi ti co empl eado se seleccionar, por supuesto, sobre l a
base del cul ti vo y del antibiograma, y cuando se i nici e de forma
emprica, habr que tener en cuenta factores que orienten haci a
el germen causante: mayor i nci denci a de Pseudomonas en perso
nas di abticas y enfermos de UVI, estafilococo en adictos a drogas
parenterales (MI R 03-04, 84), Proteus en pacientes con li ti asi s i n
fectiva, presencia de sondas, catteres, tratamientos anti bi ticos
previos, etc.
En la evaluacin del paciente con pielonefritis y mala respuesta a tra
tamiento i ni ci al , es recomendable la realizacin de una ecografa para
descartar obstruccin o litiasis (MI R 00-01 F, 134).
ITU en varones
Cualquier ITU en varn debe considerarse como complicada i nici al
mente ya que hay que asumi r que existe afectacin del tejido prost
tico, renal o que existen problemas concomitantes como obstruccin
urinaria, litiasis o malformaciones urolgicas. Por todo ello, el trata
miento debe ser ms prolongado (mnimo una semana), no siendo ade
cuados los cursos cortos de tratamiento.
Prostatitis
La i nfeccin aguda del tej ido prosttico se presenta como un cuadro
sptico con afectacin general del paciente, fiebre elevada, sndrome
miccional, artromialgias y dificultad miccional (Tabla 3). En el examen
rectal, la prstata aparece muy dolorosa e inflamada. El germen ms
habitual es E. coli. Durante la inflamacin aguda, los antibiticos pe
netran adecuadamente, pero una vez que sta cede, la penetracin es
ms pobre. Por ello, se deben utilizar cursos largos de tratamiento (3-4
semanas) para i ntentar evitar la persistencia de focos que den pie a una
prostatitis crnica. Entre los antimicrobianos empleados, las fluoroqui
nolonas son las que mejor difunden al tejido prosttico .
RECUERDA
En pacientes con SIDA, Cryptococcus neoformans puede ser una causa
de prostatitis, ya que se elimina a travs de la orina.
La prostatitis crnica bacteriana suele presentarse como molestias
perineales o genitales, sntomas i rritativos (polaquiuria, tenesmo, es
cozor) y episodios de ITU recurrentes causados por el mismo organis
mo. En el lquido prosttico se evidencian ms de 10 leucocitos por
campo de gran aumento, y macrfagos que contienen cuerpos ovales
grasos.
ETIOLOGIA ClINICA H 'ITU CULTIVO ORINA LIQUIDO PROSTATlCO
CULTIVO LIQUIDO
TRATAMIENTO
PROSTATICO
-
Cotrimoxazol,
ogudo
E.coH Cuadro sptico + + Nunca hacer masaje prosttico ni sondaje fluoroquinolonas
4 semanas
Pl Irritativo con Cotrimoxazol,
c EcoJ reagudizaciones, sin + +/
-
> 10 leucocitos/campo + fluoroqulnolonas
- febre ni leucocitosis 6-12 semanas
-
Ureaplasma Cronicidad,
crnica
- -
> 10 leucocitos/campo
-
Doxicidina
no*no
Myeoplasma empeoramiento
-Desconocida Oscilante
- -
< 10 leucocitos/campo -
a-bloqueantes
Relajantes musculares
Tabla 3. Oiagnstko difrencial de las prostatitis
5
Manual CTO de Medicina y Ciruga, B." edicin
El tratamiento debe estar guiado por los cultivos, tanto de orina como
de fluido obtenido por masaje prosttico, y prolongarse entre 4 y 1 6
semanas. Cuando se encuentra a un paciente con datos de prostatitis
crnica y signos i nflamatorios en el lquido prosttico, pero sin historia
documentada de ITU y con cultivos negativos, el cuadro se denomina
prostatitis no bacteriana. En ocasiones, el responsable puede ser U.
urealylicum o M. hominis, pudiendo ser tratados estos casos con doxi
ciclina o eritromicina, sobre esta sospecha.
Se denomina prostatodi ni a a un cuadro clnico si mi l ar donde predo
mi nan las molestias perineales o genitales con cul tivos negativos y
menos de 1 0 leucocitos por campo en el lquido prosttico. Su cau
sa es desconocida y el tratamiento difcil, emplendose actualmente
p-bloqueantes o relajantes musculares como terapia inicial.
Orqui epi di di miti s
En varones adultos menores de 35 aos es considerada, en el plano
terico, una enfermedad de transmisin sexual, siendo los agentes
ms frecuentes Chlamydia trachomatis y Neisseria gonorrhoeae (Ta
bla 4). Por enci ma de 35 aos, los mi croorgani smos ms frecuentes
son las enterobacterias. El tratamiento puede l l evarse a cabo con las
sigui entes pautas: 1 ) qui nolonas, 2) ceftriaxona en dosis ni ca i . m.
( 1 25-250 mg) ms 1 0 das de doxi ci cl i na ( 1 00 mg1 2 h/7 das), se
apl i car esta pauta en aquel l os casos en l os que se sospeche ETS
( MI R 99-00, 1 35).
Absceso renal
Los abscesos medulares o corticales suelen proceder de un foco de pie
lonefritis contiguo O de diseminacin hematgena de s. aureus/ proce
dente de focos cutneos en sujetos adi ctos a drogas por va parenteral .
El urocultivo en este lti mo caso puede ser negativo. El di agnstico
ms fi abl e se realiza mediante TC Deben tratarse con anti bi ticos
por va intravenosa y, dependi endo del tamao y de la evol uci n,
se hace obligatorio el drenaje mediante puncin percutnea O qui rr
gicamente.
Absceso pe,i"enal
Se l ocal i za entre la cpsula renal y la fascia de Gerota. Lo ms fre
cuente es que un absceso cortical se abra a este espacio, pero pue
de ocurrir tambin por di semi naci n hematgena. El germen ms
RECUERDA
S. oureus es tambin la causa ms frecuente
de endocarditis Infecciosa.
ENFERMEDAD ETIOLOGIA
Neisserio gonorrhoeoe
frecuente es E. coli, y
S. aureus en los casos
de di semi naci n he
matgena (Fi gura 1).
LESiN TIPICA
Asintomtica ()
Exudacin uretral matutina (1)
El diagnstico es si mi l ar al absceso renal, y su tratamiento pasa por
el drenaje percutneo o qui rrgico, con la adecuada cobertura anti
bitica.
ITU asociada a catteres
La ITU es la infeccin hospitalaria ms frecuente, y los catteres urina
rios la principal fuente de sepsis. Se calcula que el 1 % de cateterismos
ambulatorios transitorios sufren una ITU posterior y que la mayora de
enfermos con catter permanente presentan una bacteri uri a significati
va al cuarto da de su colocacin. Esta bacteri uria puede hacerse si nto
mtica en forma de cuadros de cistitis, hematuria o episodios febriles,
muchas veces autolimitados.
DIAGNSTICO TRATAMIENTO
Contacto < 5 dlas Ceftriaxona o
Gram de exudado cervical espectinomicina (no en
Ur gc
Epididimoprostatitis, salpingitis, sndrome Cultivo en medio de Thayer-Martin farngeas) Ciprofoxacino
Fit-Hugh-Curtis, gonococemia diseminada
(dfcit C5-C.. menstruacin, embarazo,
auxotipo AHU)
Chlomydio trochomotis, Similar a las UG, pero con menos signos Contacto 7-15 dias. Excluir gonorrea Tetraciclinas o macrlidos
lIno _ Ureoplosmo ureolticum y sntomas por Gram y cultivo. C. inclusin-
Epldidimitis, proctitis, cervicitis, EIP Giemsa IFD, medios celulares
Tabla 4. Diagnstico difrencial de las uretritis
6
Entre los factores que aumentan el ri esgo de ITU asociada a catter
urinario se pueden enumerar: 1) sexo femenino, 2) edad avanzada, 3)
mala tcnica de sondaje, 41 sistemas de drenaje abi ertos y 51 falta de
higiene local.
Entre los antibiticos disponibles, parece que las quinolonas son los
que mejor elimi nan la pelcula biolgica de los catteres i nfectados,
favoreciendo as el tratamiento de la i nfeccin; en cualquier caso, ste
nicamente se recomienda si existe si ntomatologa o en el momento de
la retirada del catter, por el mayor ri esgo de ITU sintomtica y sepsis.
2.4. Tubercul osis genitouri nari a
Generalmente est ocasionada por Mycobacterium tuberculosis. El apa
rato genitourinario es el sitio ms frecuente de afectacin extrapulmonar
(tras la adenitis tuberculosa). Un 5% de los pacientes con tuberculosis
activa presentan afectacin del tracto genitourinario (Figura 2).
Tuberculosis
miliar
Amputacin
catieial
Trompa
Estenosis
ureteral distal
Pionefrosis
Mierovejiga
Prstata
y vesfeulas
seminales
Epididimitis
Figura 2. Lesiones de la tuberculosis genitourlnaria
Tras la inhalacin del bacilo, se produce una diseminacin hematgena
(primoi nfeccin) con siembra de bacilos en ambos riones en el 90% de
los casos. Sin embargo, la enfermedad clnica generalmente es uni lateral .
El periodo de latencia entre la "siembra" y la enfermedad cl ni ca oscila
entre 1 O Y 40 aos, afectando principalmente a pacientes por debajo de
los 50 aos. La lesin i ni ci al microscpica se localiza en los glomru
los en forma de granulomas mi croscpicos. Al avanzar l a enfermedad,
se produce afectacin ms distal hasta la aparicin de una papilitis
necrotizante, momento en el cual ya puede existir paso de bacilos a l a
va excretora donde, por procesos infl amatorios, ocasionar estenosis
a nivel de los infundbulos caliciales, pelvis y urter, con hi dronefrosis
secundaria. Las lesiones renales pueden cavitarse y calcificarse, y lle
gar a producir una destruccin total del parnquima (fenmeno que se
denomina "rin mastic" ).
urologia _
Cl ni ca
Los hal l azgos cl nicos son escasos. En el 70% de los paci entes, los sn
tomas son leves. Lo ms frecuente es la aparicin de microhematuria,
dolor vago en flanco o clico renal. La afectacin vesical, sin embargo,
s produce sintomatologa florida con un sndrome cisttico rebelde,
donde l a polaqui uri a (secundaria a la disminucin de la capacidad
vesical) es lo ms l l amativo. En varones, es frecuente l a aparicin de
una orquiepididimitis crnica que no responde a la terapia habitual.
En el 90% de los pacientes, el anlisis urinario es anormal. Tpicamente
aparece piuria cida con urocultivo negativo. La prueba de laborato
rio ms importante es el cultivo de M. tuberculosis en medio selectivo
(Lowenstein), ya que los medios de tincin rpida (Ziehl, auramina),
aunque vlidos, pueden dar falsos positivos por contaminacin con M.
smegmatis (MtR 02-03, 174).
Di agnstico
El cultivo en medio de Lowenstein es positivo en el 90% de los pa
cientes con enfermedad activa, aunque deben obtenerse, al menos, tres
muestras de das diferentes para mejorar la sensibi l i dad, ya que el paso
de bacilos a orina no es constante. Actualmente, lo ms rentable es rea
l i zar una PCR de orina en busca del ARN del bacilo.
Radiolgicamente, el 90% de los pacientes presentan urogramas altera
dos. El hallazgo ms sugestivo es la presencia de cavidades que comu
nican con el sistema colector. I ni cialmente estas cavidades son mnimas
y dan un aspecto "mordisqueado" a los cli ces. Segn la enfermedad
avanza, pueden encontrarse estenosis infundibulares, ureteropilicas, en
unin ureterovesical o vejigas pequeas de aspecto rgido. En el punto
ms evolucionado de la enfermedad, el rin puede encontrarse anula
do, disminuido de tamao y con calcificaciones parenquimatosas.
Tratamiento
El tratami ento mdico de l a enfermedad activa no difiere sustancial
mente del de la tuberculosis pulmonar en cuanto a frmacos y periodo
de tratamiento. Puede ser necesario el tratamiento quirrgi co, depen
diendo de la complicacin asociada, generalmente estenosis de la va
excretora e hidronefrosis. En caso de rin no funcionante por lesin
extensa del parnquima, puede ser precisa l a nefrectoma.
*:, isoniacida y pirazinamida son los tres antibiticos ms em
pleados en la t uberculosis.
2. 5. Cisti ti s interstici al
Aunque no es un cuadro infeccioso, s e incluye en el presente captulo
esta entidad inflamatoria vesical de origen desconocido. En este senti
do, se esgrimen dos teoras no demostradas: por un lado, la teora au
toinmunitaria, y por otro, l a de un dficit en el recubrimi ento urotel i al
por glucosaminoglucanos.
7
Manual elO de Medici na y Ciruga, s.a edicin
Cl nica
Suele presentarse en mujeres entre 30 y 70 ar1os, como un cuadro cis
ttico crnico en el que destacan disuria, polaquiuria con nicturia y
dolor suprapbico, acompaados en ocasiones de hematuria (20-30%)
(MIR 01-02, 1 06).
RECUERDA
Existen muchas ms causas de sndrome cisttico: cistitis aguda, tuberculo
sis, carcinoma in situ, etc.
Di agnstico
El di agnstico es bsicamente por exclusin de otra patologa que
pueda ocasionM un cuadro si mi l ar ( i nfeccin bacteriana, tuberculosis,
8
Un prosttico, sin otros problemas de salud, portador de sonda uretral pennanente,
presenta bacleriuria (> 10 unidades formdoras de colonias) en dos uroculti\os.
Cul es la actitud teraputica ms con\lenientel
1 ) Tratamiento antibitico de amplio espectro.
2) Tratamiento antibitico segn antibiograma.
3) Cambio de sonda urinaria exclusivamente.
4) Antispticos en vejiga urinaria.
5) Cambio de sond.l urinaria y tratamiento antibitico.
RC: 3
Ante un paciente de 24 aos, que presenta fiebre alta con dolor, inflamacin y
enrojecimiento testicular izquierdo, cul de las siguientes afirmaciones e INCO
RRECTA?
1) El diagnstico ms probable es el de epididimitis.
2) los patgenos ms frecuentcs son Chlamydia lrachomalis y Neisseria gonorrhoae.
3) El tratamiento d eleccin es vancomicna + gentamicina.
4) El tratamiento de eleccin puede sr ofloxacino.
5) Un tratamiento alternativo es ceftriaxona en monooosis ms 1 o das de doxiciclina.
MIR 99-00, 135; RC: 3
Ante un paciente que presenta febrcula persistente, crisis renoureterales bre\es,
piuria estril, orina con pH cido, microhematuria persistente, con citologa uri
naria negati\a y epiddimos indurados .en qu enfermedad se debe pensar pri
mero?
1) Sarcoidasis.
2) Carcinoma vesical.
3) Carcinoma renal.
litiasis o tumor vesical) apoyado en los hal lazgos cistoscpicos suges
tivos: 1) petequias submucosas, principal mente trigonales, que apare
cen al distender la vejiga (glomerulaciones), 2) lceras de Hunner. La
biopsia vesical, adems de descartar la presenci a de carcinoma in situ
u otra patologa, revela en algunos casos, un infi ltrado intersticial de
mastocitos (MIR 00-01 F, 1 45).
Tratamiento
Aunque esta enfermedad raramente supone una amenaza para la vi da de
la paciente, su morbilidad es elevada. Desgraciadamente, las diversas al
ternativas de tratamiento ni camente pueden encaminarse a una mejora
si ntomtica, en la mayora de los casos con resultados discretos; 1 ) dis
tensin hi drulica vesical, 2) amitript i l i na oral, 3) instilacin con dime
tilsulfxido [DMSOJ, 4) corticoides tpicos o si stmicos, 5) denervacin
vesical, 6) cistoplastias e aumento, y en ltimo trmino, 7) cistectoma.
Casos clnicos representativos
4) TBe urogenital.
5) Pielonefritis crnica por P. aeruginosa.
RC: 4
Una paciente de 27 aos acude al srvido de Urgencias por dolor en fosa renal
derecha, fiebre de 39 ", escalofros y sndrome micdonal acompaante. Es alrgica
a penicilinas. Seale la respuesta correcta:
1 ) No ser necesario descartar patologa urinaria obstrUd;va en este caso, ya que
presenta un claro sndrome miccional.
2) Para poder hacer el diagnstico de pielonefritis se deber conocer primero 10$
datos referidos a la funcin renal.
3) Se deber iniciar tratamiento emprico con un -Iactmico.
4) Si en las primeras horas evoluciona favorablemente podr continuar el tratamien
to de forma ambulante.
5) El mejor tratamiento disponible es la administracin intramuscular d aminoglu
csidos.
RC: 4
Un paciente de 83 aos sondado de forma permanente acude a la consulta tras
detectrsele dos cultivos positivos lomados con una semana de diferencia. Asegura
encontrarse asintomtico. la acttud ms adecuada ser:
1) Iniciar tratamiento antibitico segn antibiograma de los cuhivO obtenidos.
2) Tranquilizar al paciente y seguir con su pauta habitual de recambio de sonda.
3) Realizar cambio de sonda d forma inmediata con tratamiento antibitico.
4) Realizar cambio de sonda de forma inmediata con profila)is antibitica de 4 das.
5) Retirar la sonda y colocar cistotoma suprapbica.
RC: 2
0J.
UROLlTIASIS
OrientaCIn
MIR
Tema fundamental en
esta asignatura. Se debe
conocer muy bien la actitud
ante la litiasis en general,
ante los distintos tipos de
clculos y, especialmente,
todo lo relacionado con
el tratamiento. Es un tema
rentable y agradecido. as que
hay que emplear el tiempo
necesario. la tabla-resumen
de urolitiasis puede ser de
gran ayuda.
[ Preguntas
-MIR 08-09, 93
-MIR 06-07, 93, 106
-MIR 05-06, 104
-MIR 04-05, 104
- MIR 03-04, 44, 80
-MIR 02-03, 1 76
- MIR OO-01, 1 1 7
- MIR 99-00, 1 89
- MIR 99-00F, 143, 146
- MIR 98-99, 135, 138
- MIR 98-99F, 144
O
II
m
Los clculos ms frecuentes son los de oxalato clcico.
Globalmente, la litiasis es ms comn en el varn, salvo las de estruvita, ms comunes en mujeres.
La radiografa de abdomen no permite ver algunos clculos, como los de urato. Sin embargo, la ecografa
puede verlos, independientemente de su composicin.
Litiasis radiotransparentes: Sulfamidas, Indinavir, Urato, Xantinas (SIUX). las de cistina son radiolcldas; y
el resto, radioopacas.
las tiazidas son tiles para la hipercalciuria idioptica.
los clculos asociados a las resecciones ileales O a la enfermedad inflamatoria intestinal son de oxalato
clcico.
Precipitan en medio cido: cido rico y cistina. Precipitan en medio alcalino las que contienen fosfatos
(fosfato amnico magnsico o estruvita, y el fosfato clcico).
En el tratamiento de la litiasis por cido rico es beneficioso alcalinizar la orina.
los clculos de oxalato NO se ven alterados por el pH (al Oxal, el pH de la igual).
los clculos de estruvita se relacionan con microorganismos productores de ureasa, como Proteus.
las contraindicaciones absolutas para la LEOC son: embarazo, infeccin activa y obstruccin de las vas
urinarias distal al clculo.
3. 1 . Epidemiol oga
Son numerosas las sustancias que se han identificado formando parte de 105 clculos. Su i ncidencia vara segn
el pas, e incluso segn las reas geogrficas dentro del mismo pas.
Se pueden distinguir seis grupos de componentes:
Oxalato clcico.
Fosfato clcico.
Fosfato no clcico.
Compuestos purnicos (cido rico, urato amnico, urato sdico, xantina, 2, 8 dihidroxiadenina).
Ami nocidos (cistina).
Otros (carbonato clcico, sulfamidas, etc.).
Los clculos de oxalato clcico son los ms frecuentes, con cifras en torno al 65%, seguidos por los infectivos
y cido rico (alrededor del 15% cada uno). Fosfato clcico un 5%, y los de cistina con una incidencia baja
(1-3%).
La tercera dcada es la edad media de aparicin, por primera vez, de la litiasis salvo en los de cistina, que suelen
ser de aparicin ms prematura.
En Espaa, l a incidencia de litiasis alcanza al 4,2% de la poblacin, con mayor afectacin de varones que mu
jeres.

nicamente los clculos infectivos tienen mayor incidencia en l a mujer.


9
0J.
UROLl TIASIS
OrientaCin
MIR
T er fundamental en
esta asignatura. Se debe
conocer muy bien la actitud
ante la litiasis en general.
ante los distintos tij de
clculos y. especialmente,
todo lo relacionado con
el tratamiento. Es un tema
rentable y agradecido, as que
hay que emplear el tiempo
necesario. La tabla-resumen
de urolitiasis puede ser de
gran ayuda.
U Preguntas
- MIR 08-09, 93
- MIR 06-07, l3. 106
- MIR 05-06, 104
- MIR 04-05, 104
- MIR 03-04, 44, 80
- MIR 02-OJ, ' 76
- MIR OO-Ol , 1 1 7
- MIR 99-00, 189
- MIR 99-00F, 143, 146
- MIR 98-99, 135, 1 38
- MIR 98-99F, 144
o
Aspectos esenciales
los clculos ms frecuentes son los de oxalato clcico.
Globalmente, la litiasis e ms comn en el varn, salvo las de estruvita, ms comunes en mujeres.
la radiografa de abdomen no permite ver algunos clculos, como los de urato. Sin embargo, la ecografa
puede verlos, independientemente de su composicin.
Litiasis radiotransparentes: Sulfamidas, Indinavir, Urato, Xantinas (SIUX). las de cistina son radiolcidas; y
el resto, radioopacas.
las tiazidas son tiles para la hipercalciuria idioptica.
los clculos asociados a las resecciones ileales o a la enfermedad inflamatoria intestinal son de oxalato
clcico.
Precipitan en medio cido: cido rico y cistina. Precipitan en medio alcalino las que contienen fosfatos
(fosfato amnico magnsico O estruvita, y el fosfato clcico).
En el tratamiento de la litiasis por cido rico es beneficioso alcalinizar la orina.
los clculos de oxalato NO se ven alterados por el pH (al Oxal, el pH de la igual).
los clculos de estruvita se relacionan con microorganismos productores de ur easa, como Proteus.
Las contraindicaciones absolutas para la LEOC son: embarazo, infeccin activa y obstruccin de las vas
urinarias distal al clculo.
3. 1 . Epi demiol oga
Son numerosas las sustancias que se han identificado formando parte de los clculos. Su i nci denci a vara segn
el pas, e incluso segn las reas geogrficas dentro del mismo pas.
Se pueden distinguir seis grupos de componentes:
Oxalato clcico.
Fosfato clcico.
Fosfato no clcico.
Compuestos purnicos (cido rico, urato amnico, urato sdico, xantina, 2,8 di hidroxiadenina).
Aminocidos (cistina).
Otros (carbonato clcico, sulfamidas, ete.).
Los clculos de oxalato clcico son los ms frecuentes, con cifras en torno al 65%, seguidos por los i nfectivos
y cido rico (al rededor del 1 5% cada uno). Fosfato clcico un 5%, y los de cistina con una i ncidencia baja
( 1 -3%).
La tercera dcada es la edad media de aparicin, por primera vez, de la li ti asi s salvo en los de cistina, que suelen
ser de aparicin ms prematura.
En Espaa, la incidencia de l i tiasis alcanza al 4,2% de l a poblacin, con mayor afectacin de varones que mu
jeres.

nicamente los clculos i nfectivos tienen mayor incidencia en l a mujer.


9
Manual eTO de Medicina y Ciruga, B.a edicin
RECUERDA
las infeciones urinarias son ms frecuentes en mujeres que en varones.
Por e los clculos d etruvita tambin lo son.
La enfermedad litisica recidiva en el 40% de los casos, con una media
de un nuevo clculo cada dos o tres aos. Por recidiva se entiende l a apa
ricin de una nueva l itiasis de la misma composicin y en la misma lo
calizacin, en un intervalo menor de cuatro aos entre un clculo y otro.
3. 2. Manifestaci ones cl ni cas
ysu manejo agudo
El dolor agudo del clico renal es l a mani festacin ms tpica de la l i
tiasis renal. El dolor se produce por l a sobredistensin de l a va urinaria
tras l a obstruccin de sta por el clculo. Es lgi co, por tanto, que el
clculo deba desplazarse desde su origen cal i ci al para producir sinto
matologa aguda. Ocasionalmente se observan cuadros de dolor vago
renal en relacin con litiasis caliciales no desplazadas.
El clico renal o crisis renoureteral suele aparecer de forma progresi
va sobre la fosa lumbar afectada, irradi ndose por el flanco hacia la
i ngl e y los genitales (Figura 3). El paciente generalmente se encuentra
afectado, con dolor que no cede con reposo, por lo que cambia de
postura continuamente. Puede acompaarse de un cortejo vegetativo
con nuseas, vmitos y sudoracin. El dolor irradiado hacia la i ngl e
generalmente i ndi ca que el cl cul o ha al canzado el urter. Cuando se
encuentra en vecindad de la vejiga, o bi en dentro de sta, puede apare
cer un cuadro irritativo, si mi l ar al sndrome miccional con polaquiuria,
disuria y tenesmo vesical.
Sudoracin
Dolor de inicio
Dolor irradiado
RX simple de abdomen
con clculo a nivel de l
Sndrome miccional
Hematuria
Figura 3. Diagnstico de urolitiasis
Los clculos i nfectivos de estruvita, y en menor medida, los de cido
rico y cistina pueden crecer modelando las cavidades renales ( l itiasis
1 0
coraliforme o "en asta de venado" [Figura 4]), manifestndose no como
clico, sino como infecciones urinarias de repeticin, dolor lumbar
sordo, hematuria o i ncl uso insuficiencia renal terminal.
Di agnstico
El anl i si s bsico de orina muestra generalmente hematuria y leuco
cituria. Una pi uri a i mportante apoyara l a posi bi l idad de infeccin
sobreaadida, aunque ni nguno de estos datos es realmente determi
nante.
Los cristales de oxalato clcico di hidralado aparecen como bi pi r
mi des tetragonales al observarlos con lupa bi nocular. Los de oxa
lato clcico monohi dratado aparecen como cristales al argados que
adoptan forma de empalizada, formando cl cul os de estructura ra
diada, con aspecto compacto y macizo. Entre los fosfatos clcicos,
la brushi ta es el compuesto ms cido, formando cristales grandes
en forma de abanico de color azul con luz polarizada. Las apatitas
tienen aspecto microgranul ar o esferoctico. El cido rico aparece
bajo la lupa como una desordenada aglomeracin de cristales. En
algunos clculos, los cristales estn tan j untos que se asemejan a una
masa continua.
La estruvita (o fosfato amnico magnsico) es el componente ms ca
racterstico de los clculos producidos por infeccin por grmenes
urealticos. Sus cristales tienen formas prismticas polimorfas, y rara
mente se observan los cristales "en atad" que pueden hal larse en el
sedimento. La cistina se reconoce fcil mente por su aspecto acarame
lado, formando cristales hexagonales en prismas o lminas.
En teora, el 90% de los clculos son visibles en una radiografa simple
de abdomen, aunque este porcentaje es considerablemente menor en
las radiografas urgentes sin preparacin intesti nal . Radiolgicamente,
la mayora de los clculos son radioopacos, exceptuando los de cido
rico y algunas otras composiciones infrecuentes (sulfamidas, xantina,
i ndi navir) (MIR 04-05, 1 04).
El estudio de imagen se completar medi ante otras tcnicas di ag
nsticas. La ecografa permi ti r vi sual i zar i ncl uso las l iti asi s radio
transparentes, con el inconveniente de no ser vistas aquel l as ubi
cadas en el trayecto ureteral (salvo las zonas cercanas a l a vejiga o
al ri n). Tambi n se podr evaluar el grado de hi dronefrosis ( MI R
99-00, 1 89).
La urograffa ofrece informacin morfolgica y funci onal de ambos
riones (Figura 5). Debe tenerse en cuenta que, durante el clico
renal, puede observarse una anul aci n funcional, sin que si gni fi
que necesariamente deterioro de dicha uni dad renal. Mediante
esta tcnica se puede diagnosticar todo tipo de clculos, ya sean
radiotransparentes o radioopacos. El pri nci pal i nconveni ente de
este procedi miento es l a introduccin de contraste yodado, que est
contrai ndi cado en los pacientes con alergia, creati ni na mayor de 2,
mi el oma ml ti ple o deshidratacin i mportante.
Segn las guas clnicas, la urografa i ntravenosa (UIV) actualmente ha
sido desplazada por la Te helicoidal sin contraste, que se ha convertido
en el nuevo estudio de referencia para las litiasis. Aunque su alto coste
hace que todava no est extendido su uso, permite evaluar todo tipo
de clculos.
Tratami ento
El manejo agudo del cl i co renal se basa en el control del dolor.
Para esto, es preciso conseguir una di smi nuci n de l a presin den
tro de l a va uri nari a, J o que puede hacerse, sobre todo, con anti
i nfl amatorios, que di smi nuyen el dol or y l a di uresis al i nhi bi r l a
sntesis de prostagl andi nas. Asi mi smo, se pueden usar espasmolti
cos, que di smi nuyen l a presin intraureteral al relajar l a pared del
urter.
Existen una serie de situaciones en las que el clico renal se convierte
en una urgencia que precisa de hospitalizacin, y eventualmente, de
manipulacin invasiva:
Obstruccin grave, principalmente si se acompaa de litiasis mayor
de 1 0 mm.
Fiebre elevada (mayor de 38 OC).
Dolor incontrolable.
Rin nico.
Asi mi smo, en pacientes di abticos, por el mayor ri esgo de com
pl icaciones, es aconsejable, si no el i ngreso, al menos una obser
vacin estricta. Una situacin si mi l ar ocurre durante el embarazo,
donde una di l atacin leve de l a va uri nari a puede considerarse
" fisiolgica", pero obstrucciones ms i mportantes o la apari ci n
de fiebre hacen aconsejable l a colocacin de un catter ureteral
(Figura 6).
.
Figura 6. Doble J derecho. litiasis ureteral derecha. litiasis coraliforme izquierda
3. 3. Eval uaci n ytratamiento
de l a l iti asis renal
Este apartado se puede di vi di r en dos partes. Por u n lado, el estudio
de la l i tiasi s con l a fi nali dad de instaurar un tratamiento preventivo
de su formacin, y por otro, el estudio y tratamiento de la l i tiasi s ya
formada.
Estudio
y tratamiento preventivo
La evaluacin del paciente con l i ti asi s se basa en un estudio meta
blico para determi nar qu factores son modificables, en un intento
de evitar l a recidiva (Tabla 5).
Este estudio debe reservarse para aque
l l os pacientes con alta probabi l i dad de
recidiva, aunque cada vez ms autores
i ndi can que debe reali zarse a todos Jos
pacientes (Tabla 6).
RECUERDA
la furosemida, al re
v que las tiazidas,
aumenta el calcio uri
nario.
1 1
Manual CTO de Medicina y Ciruga, B.a edicin
LITIASIS CALCICA LITIASIS LITIASIS
LITIASIS
(OXALATOO FOSFATO) URICA CtSTINICA
INFECCIOSA
Hipercalciuria idioptica
Hipercalciuria secundaria
a hiprcalcemia
Hiperuricosuria
Hiperoxaluria
Hipocitraturia
Acidosis renal tubular
distal
litiasis clcica idioptica
Gota primaria
Hemopatas
Enfermedades
digestivas
Ingesta excesiva
de purinas
Frmacos
litiasis rica
idioptica
. Cistinuria
(ESTRUVITA)
. Infecciones
por grmenes
productores
de ureasa
Tabla 5. Tipos de litiasis. Situaciones que favorecen su aparicin
Edad temprana deaparicin
Litiasis bilateral
litiasis en rin nico o malformado
Composicin poco frecuente
litiasis recidivante
Nefrocalcinosis
Litiasis coraliforme
Tabla 6. Pacientes con indicacin de estudio metablico
Desde el punto de vista prctico, las litiasis se pueden divi di r en: las
de composicin clcica y las de otras composiciones, ya que el primer
grupo supone la mayora de los casos (70-80%) tratados habitualmente.
Litiasis clcica
En la mayora de las ocasiones se desconoce el origen de la litiasis cl
cica, aunque se puede hacer una aproximacin a los factores de riesgo
que influyen en su aparicin. Slo en un pequeo porcentaje de casos
existe una enfermedad de base que puede ser tratada, y de esta forma
desaparece la formacin de clculos clcicos.
Hipercalciuria idioptica. Es la causa ms frecuente de litiasis cl
cica. Se define como una excrecin urinaria de calcio mayor de
300 mgl24 h en el varn y 250 mgl24 h en la mujer. De cara a su
manejo, las tiazidas di smi nuyen el calcio urinario, reduciendo la
formacin de litiasis (MIR 03-04, 44; MIR 99-00F, 1 46; MIR 98-
99, 1 35). La admi nistracin de citrato potsico ayuda a evitar la
hi popotasemia y aumenta el citrato urinario, que es i nhibidor de la
l i tognesis (Tabla 7).
I
ABSORTIVAS RESORTIVAS RENALES
,
Aporte excesivo
Sd. de Surnen (leche
y alcalinos )
Hipervitamlnosls O
Idioptica
Sarcoidosis
Hiperparatiroidismo
Inmovilizacin
Tumorales
Enf. Paget
Sd. deCushing
Acidosis tubular
distal
Idioptca
Tabla 7. Cuss ms fe d hiprcakiuria
1 2
Hiperuricosuria. Excrecin en orina de ms de 800 mgl24 h en el
varn o 750 mgl24 h en la mujer. Adems de favorecer la litiasis
rica, l a hiperuricosuria constituye un factor de riesgo para la for
macin de clculos de calcio, probablemente por nucJeacin hete
rognea sobre ncleos de cido rico o urato sdico. General mente
se debe a un exceso de purinas en l a di eta.
Hiperoxaluria. Se considera como tal la excrecin en orina de ms
de 40 mgl24 h. Existe una hiperoxaluria primaria, que es conse
cuencia de un defecto enzimtico autosmico recesivo; no tiene
tratamienlo y generalmente conduce a insuficiencia renal por litiasis
recidivante. El nico tratamiento que existe actualmente es el tras-
plante heptico, que suele ir unido al renal, aunque algunos casos
responden a piridoxina.
RECUERDA
La causa ms frecuente de hipercalcemia en un paciente ambulatorio
e el hiprparatiroidismo primario. En cambio, la hipercalcemia ms
frecuente en uno in
g
resado es la de origen neoplsico.
No obstante, l a mayora de los casos de hi peroxaluria son secun
darios a malabsorcin de cidos grasos por enfermedades crnicas
pancreatobi l i ares, derivacin intestinal para el tratami ento de la
obesidad mrbida, reseccin ileal, enfermedad inflamatoria intes
tinal (MIR 05-06, 1 04; MI R 00-01 , 1 1 7), hipercalciuria coincidente
o por falta de calcio en la di eta, lo que permite que exista mayor
cantidad de oxalato i ntesti nal para su absorcin.
La intoxicacin por etilenglicol y metoxiflurano puede producir hi
peroxaluria, as como la i ngesta de vitamina C en altas dosis. En
todos estos casos secundarios, el tratamiento con colestiramina, una
dieta pobre en grasas y l a correccin de l a malabsorcin, en l a me
dida de lo posible, suelen ser medidas eficaces.
Hipocitraturia. Excrecin de citrato inferior a 300 mgl24 h. Gene
ralmente se asocia a otras anomalas urinarias. Aunque de causa
desconocida, puede contri bui r una di eta rica en protenas, hi poca
liemia, enfermedad intestinal o infeccin urinaria.
Hiperparatiroidismo primario. Supone la causa ms frecuente de
hipercalciuria conocida (vase Seccin de Endocrinologa, metabo
lismo y nutricin) (MIR 98-99F, 1 44) .
Acidosis tubular renal distal (vase Seccin de Nefrologa). Enfer
medad autosmica recesiva. Consiste en la imposibilidad del tbulo
distal para excretar hidrogeniones a l a orina (orinas persistentemen
te al cal i nas) con aumento de l a el i mi nacin de calcio a l a orina.
Existen formas i ncompletas que se observan en pacientes formado
res de clculos de oxalato clcico y con hipercalciuria idioptica.
En stos, probablemente la acidosis tubular no juegue un papel i m
portante y responden a tiazidas (MI R 98-99, 1 38).
Otras circunstancias que favorecen la litiasis clcica son: sarcoido
sis, sndrome de Cushi ng, di uresis escasa, dficit de inhibidores o
anomalas en el pH urinario (alcalosis).
Litiasis clcica idioptica. Aproximadamente en el 20% de los pa
cientes con litiasis clcica no se demuestra ninguna anomala en el
estudio metablico.
Litiasis rica
El cido rico no disociado es poco soluble en orina. Con un pH uri
nario de 5, la solubil i dad del cido rico es nicamente de 1 00 mgl,
mientras que con un pH de 7 es de 1 .580 mgll. Esto demuestra l a gran
importancia del pH urinario en l a formacin de clculos de cido ri
co. Aparte de stos, tambin existe una pequea proporcin de clcu
los de urato monosdico y urato amnico.
El objetivo del tratamiento es reducir el cido ri co excretado y au
mentar el pH uri nari o (MI R 03-04, 80), ya que los clculos ms fre
cuentes en pacientes hiperuricmicos son los de cido rico. Por otra
parle, este tipo de cl cul os son los que mejor responden al tratami en
to mdi co mediante qui mi l i si s por al cal i nizacin uri nari a. Pueden
admi ni strarse di versos l cal i s; el citrato potsico impedira el teri
co ri esgo de formacin de clculos clcicos por su efecto i nhi bidor,
pero tambin pueden tratarse con bicarbonato sdico o ci trato sdi
CO (MI R 02-03, 1 76). Una al ternativa es l a acetazol ami da en dosis de
250 mglda. Cuando, adems, la uri cemi a es alta, puede tratarse con
al opuri nol (MI R 06-07, 93).
Litiasis cistnica
La cistinuria es un trastorno autosmico recesivo en el que existe un
defecto de absorcin, a nivel intestinal y tubular proximal, de los ami
nocidos dibsicos: cistina, ornitina, lisina y arginina (COLA), aunque
parece que puede existir un trastorno en el que nicamente se ve afec
tada la cistina, lo que i ndicara que, adems de un mecanismo de trans
porte comn, existe uno independiente para la cistina.
Los ni veles de cistina en orina de 24 horas son superiores a 1 00 mg, de
hecho, los homoeigotos pueden excretar ms de 600 mglda. El diag
nstico se realiza identificando los caractersticos cristales hexagonales
en orina, o por una prueba positiva de ni troprusiato sdico (l a orina se
tie de azul en pacientes afectados por esta enfermedad: test de Brand).
El tratamiento consiste en aumentar l a di uresis diaria (ms de 3 I/da),
alcalinizar la orina por encima de 7,5 y, en caso de que esto sea insufi
ci ente, puede i niciarse tratamiento con D-penici l amina (250 mg6 h) o
a-mereaptopropionilglieina (250 mgl6 h).
Litiasis infectiva
Los clculos infectivos de estruvita o de fosfato amnico magnsico (Mg
NHl04-6H20) se desarrol lan en un ambiente alcalino, producido por
infeccin persistente de grmenes que hidrolizan la urea, aumentando
la cantidad de amonio urinario. Los principales grmenes que poseen
ureasa, adems de diversas especies de Proteus (MIR 06-07, 1 06). son
Pseudomonas, Klebsiella, Serratia y Enlerobacter. La presencia de cuer
pos extraos (sondas vesicales, suturas) favorece su formacin.
Para su tratamiento se han empleado diversos mtodos, generalmente
ineficaces. La antibioterapia nicamente mantiene estril la orina du
rante los cursos de tratamiento.
Parece ms prometedor el uso de inhi bidores de la ureasa con cidos
hi droxmicos. Estos son molculas anlogas a l a urea que forman un
urolOgia _
complejo enzima-inhibidor irreversible. Se uti l i zan bsicamente dos
sustancias de esta naturaleza: el cido propinico y el acetohidroxmi
co. Su empleo suele venir acompaado de cefaleas, temblores, trom
bosis venosas u otros sntomas neurolgicos, por lo que tampoco son
de gran aceptacin.
Todo lo relativo al estudio de la nefrolitiasis expuesto anteriormente se
puede repasar en la Tabla 8.
Tratamiento de la l i ti asi s ya formada (Figura 7)
Los cl cul os ya formados no expulsables (> 4-5 mm) precisan de
tratami ento "agresivo", es decir, necesitan ser extrados qui rrgica
mente o fragmentados de forma que puedan ser expulsados espon
tneamente.
A continuacin se analizan brevemente las diversas formas de tratamiento.
Ciruga. Ha sido el tratamiento estndar hasta la aparicin de la
litotricia extracorprea. An hoy da, es preciso recurri r a la ciruga
cuando fracasan las ondas de choque o en determinados casos para
reducir la masa litisica (clculos coraliformes).
Endourol oga. La manipulacin endoscpica de la va urinaria es
cada da ms accesible gracias a las mejoras tcnicas. Puede reali
zarse extraccin directa del clculo mediante diversos tipos de pin
zas o cestillas, o bien fragmentar previamente el clculo mediante
diversas fuentes de energa, como l a electrohidrulica, ultrasnica o
lser. Se puede acceder hasta el clculo mediante ureterorrenosco
pia ( URS) o nefrolitotoma pereutnea ( NLPC).
litotricia extracorprea por ondas de choque (LEOC). Las ondas de
choque se transmiten a travs de los tej idos corporales con la misma
impedancia acstica que el agua hasta alcanzar l a litiasis, sobre la
que produce fenmenos de compresin y descompresin que con
ducirn a su fragmentacin.
Prcticamente todos los clculos son susceptibles de tratamiento me
diante LEOe. La nica l i mitacin seran aquellos clculos no localiza-
SALES CALCICAS
ESTRUVITA (FOSFATO
AClDO URICO CISTlNA
AMNICO MAGNSICO)
Frecuencia
. Oxalato clcico: 55-60% 10-15% 5-10% 1 -3%
Fosfato clcico: 1 0-' 5%
Sexo Varn Mujer Varn Varn =Mujer
Hipercalciuria idioptica Infeccin por grmenes ureasa (+) Gota (50%) Cistinuria
Etiologia Idioptica . Idioptica 50%)
Hiperuricosuria (20%) . Hiperuricemias secundarias
pH Alcalino Alcalino cido cido
Radiologia Radioopacos Radioopacos Radiotransparentes Radiolcidos
:iD
Formas prismticas polim6rfcas Aglomerados de cristales Cristales hexagonales en prismas
Cristales -en atad- desorganizados, a veces formando o lminas
masas continuas

Moologa
Cristales de fostato Ca
q
&o0
WO
de los cristales
1 -

1
O r
O D

f
Cristales de OxCa Cristales de estruvita Cristales de cido rico Cristales de cistina
Hipercalciuria idioptica: tiazidas cido propi6nico y cido Alcalinizar la orina Forzar diuresis (ingesta hdrical
Tratamiento
Hiperoxaluria 1 .11; piridoxina acetohidroxmico Alopurinol (si hay hiperuricemia) Alcalinizar orina
Hiperoxaluria 2.1; colestiramina Antibioterapia Dieta de bajo contenido D-penicilamina (si no hay
En ocasiones ciruga proteico respuesta)
Tabla 8. Tabla-resumen de las nefolitiasis
1 3
Manual CTO de Medicina y Ciruga, s.a edicin
1 4
MANEJO DE LAUROLlTIASIS
Crisis
CUADRO AGUDO
Estable
CUADRO CRNICO
de su tamao, composicin y
dureza, localizacin, particula
ridades anatmicas de l a va ex
cretora y paciente, funcin renal
y tipo de l i totriptor disponible.
/
- -
No comlicado Complicado
TTO. SINTOMATICO
Espasmolticos
y antiinflamatorios
Reposo e hidratacin
Obstruccin grave
Infeccin, febre
Dolor incoercible
Rin nico
Ecografa renal
Dilatacin
INGRESO YTTO. AGRESIVO
DRENAJE
(catter o nefrostoma)
no. PARENTERAL
Antibiticos
Remontar hemodinmica
Equilibrio electroltico
Narcticos
VIGILANCIA ESTRECHA
Indican o
no LEOC o
ciruga
Tratar la condicin
preexistente
l
CALCICA
acidifcar
(no til si oxalato),
citratos, tiazidas
colestiramina
y dieta baja en
grasas y rica
en calcio, si
hiperoxaluria
RICA
alcalinizar,
alopurinol
ESTRUVITA
acetohidroxmico
CISTINA
O-penicilamina,
vit. B y alcalinizar
Edad?
Tipo de clculos?
Periodicidad de la clnica?
Tipo de sntomas?
Viabilidad renal?
litotricia
t
Ciruga
t
EXTRACORPREA URETEROTOMIA
(lEOCJ PIELOllTOTOMIA
PERCUTANEA NERECTOMIA
ultrasonogrfca
con microlumbotoma
ENOOSCPICA
va ureteral
CONTRAINDICADA EN
EMBARAZO
INFECCIN
OBSTRUCCIN DISTAL
Aneurismas
Coagulopatfas
Obesidad
Arritmia cardaca
Figura 7. Manejo de la litiasis renal ya formada
La presencia de hi pertensin
arterial no controlada facilita
el riesgo de hemorragia duran
te l a sesin de l i totricia, luego
deber ser estabi l izada previa
mente a l a mi sma y constituye,
en cierto modo, por ello, con
traindicacin relativa de LEOC
(MIR 08-09, 93; MI R 99-00F,
1 43) (Tabla 9).
Complicaciones
La expulsin de fragmentos I i
tisicos puede ocasionar un
clico renal y, con menor fre
cuencia, obstruccin ureteral
(sleinstrasse o "calle l itisica/l).
Esta posi bi l i dad es mayor ante
l i ti asi s de gran tamao, por lo
que en algunos de estos casos
se puede col ocar un catter
de derivacin uri nari a (ne
frostoma o doble J) antes de
bIes por su pequeo tamao 2-5 mm). Cualquier l itiasis podra ser
tratada con LEOC, aunque esto tendr que ser matizado en funcin
la LEOC para di smi nui r este riesgo, generalmente en l i ti asi s supe
riores a 2 cm.
ABSOLUTAS
RElATIVAS (PRECISAN DE CONTROL PREVIO
Embarazo
Obstruccin distal
Infeccin activa
AL TRATAMIENTO)
Alteraciones de la coagulacin
Aneurisma artico
Alteraciones del ritmo cardaco, marcapasos
o desfbriladores
Obesidad
Hipertensin arterial descontrolada
Tabla 9. Contraindicaciones de la LEO (MIR 90 , 143)
A un hombre de 29 aos, con antecedentes de dolor tipo clico en fosa renal izquierda
que cedi con tratamiento analgsico, se le practica una urografa intravenosa, apre
cindose defcto de replecin radiotransparente d 6x7 mm en tercio distal d urter
izquierdo. El pH de la orina fue de 5,5; asimismo, se observan cristales d urato, 9-12
hemates por campo y escsa leucocituria. Cul sera el tratamiento ms apropiadol
1 ) Alopurinol va oral.
2) Ureteroscopia con extraccin del clculo.
3) Nefrolitotoma endosc6pica percutnea.
4) Alcalinizacin de la orina por va oral.
5) Administracin de D-penicilamina.
MIR 03-04, 80; RC: 4
Mujer de 50 aos, diabtia insulinodependiente, con infecciones urinarias y clicos
nefrticos de repeticin. Acude a Urgencias por dolor en fosa renal izquierda de cinco
Derivadas del efecto di recto de las ondas de choque, pueden aparecer
contusiones renales manifestadas como hematuria, hematomas rena
les, equimosis o eritema cutneo, y en grado mxi mo, rotura renal. La
hematuria se considera la complicacin ms frecuente de l a l itotricia.
Ms controvertida es la terica relacin de la LEOC con la aparicin de
hi pertensin arterial, ya que no est demostrada en las ltimas revisiones
publ icadas, aunque s la relacin entre hematoma renal post-LEOC e
hi pertensin arterial.
das de evolucin, asociado en las ltimas 24 horas a fiebre, escalofros y malestar
general. Analtica de sangre: plaquetopenia, leucocitosis y disminucin de la activi
dad de la protrombina. Analtica de orina normal. Radiografa de abdomen con claras
imgenes de litiasis. Eco renal: dilatacin moderada de sistema excretor izquierdo.
Cul es la conducta ms adecuada?
1 ) Solicitar hemocultivos y urocultivo para establecer la necesidad de antibiote
rapia.
2) Realizar urografa intravenosa para intentar filiar la caus.
3) Hidratar a la paciente bajo observacin rigurosa, y repetir ecografa a l as 48
horas.
4) Colocar catter doble) o practicar nefrostoma percutnea de forma inmediata
con cobertura antibitica.
5) laparotoma exploradora para objetivar causa, y realizar tratamiento antes de que
el cuadro est muy evolucionado.
RC: 4
04.
TUMORES RENALES
OnentaCln
MIR
El aclenocarcinoma fenal es un
tema "de moda" en el examen
MIR. Cualquier aspecto de
este lema puede aparecer,
pero es fundamental reconocer
el sndrome de $Iauffer,
que ha sido recientemente
introducido y preguntado
varias veces consecutivas.
I Preguntas
MIR 09-10, 99
- MIR 06-07, 102
- MIR 05-06, 105
- MIR 04-05, 105
- MIR 01-02, 109
- MIR 00-01 , '20
- MI R 99-00, 177
- MIR 99-00F, 144
- MIR 98-99F, 146
(
l
G
Aspectos esenciales
El ms frecuente de los tumores slidos renales es el hipernefroma.
El paciente caracterstico es un varn de mediana edad, obeso y fumador.
la trada clsica consiste en hematuria, dolor y masa en flanco, actualmente, lo ms habitual es que sea
incidentaloma (asi ntomtico). Si produce sntomas, el ms frecuente es la hematuria.
Hay que sospechar tumor renal ante un varicocele izquierdo, de aparicin sbita y que no cede con el
decbito.
El hipernefroma puede producir multitud de sndromes paraneoplsicos. Esto puede complicar bastante el
diagnstico, de ah el sobrenombre de "tumor del internista".
La elevacin de las transaminasas sin afectacin heptica es tpica del hipernefroma (sndrome de Stauffer).
No se debe confundir un quiste simple con un hi pernefroma. los criterios de quiste simple son: contorno
liso, contenido transnico y refuerzo posterior.
la primera prueba de imagen, ante la sospecha de hipernefroma, sera la ecografa.
El tratamiento fundamental del hipernefroma es la extirpacin quirrgica. La quimioterapia y radioterapia
tienen un papel muy secundario.
4. 1 . Carcinoma de cl ul as renal es
(adenocarcinoma renal , hipernefroma)
Es el tumor slido renal ms frecuente (90%) (Figura 8) (MIR 99-00, 1 77). Es un tumor fundamental mente de l a
edad adulta, con mayor incidencia entre los 40 y 60 aos, con predomini o en el varn 2: 1 a excepcin de la
variedad cromfoba, tpica de las mujeres.
Entre los factores de riesgo que se han i mpl i cado se encuentran el humo del tabaco, el cadmio y la obesidad.
Existen formas fami l i ares que suelen ser ml tiples y bi laterales, como en la enfermedad de Von Hippellindau y,
en menor medida, la esclerosis tuberosa (MIR 0001 , 1 20). Se han identificado alteraciones cromosmicas que
implican al cromosoma 3.
RECUERDA
la esclerosis tuberosa y la enferme
dad Von Hi ppel.lindau se asocian
tambin a otro tumor renal, el an
giomiolipoma.
Presentacin
Asimismo, exi ste una i nci dencia aumentada en el rin poliqustico,
en la enfermedad qustica adquirida de la insuficiencia renal crnica
y en los riones mal formados, como el rin "en herradura". Procede
de las clulas del tbulo contorneado proximal, y microscpica mente
predomi nan las clulas claras sobre las granulares y fusiformes .
La trada clsica: hematuria dolor y masa en el flanco ocurre nicamente en el 1 0% de los casos y, cuando se
presenta as, generalmente se trata de una enfermedad avanzada.
1 5
Manual eTO de Medicina y Ciruga, B.a edicin
1 4
MANEJO DE LA UROllTIASIS
Crisis
CUADRO AGUDO
Estable
CUADRO CRNICO
de su tamao, composicin y
dureza, localizacin, paricula
ridades anatmicas de la va ex
cretora y paciente, funcin renal
y tipo de l i lotriptor disponible.
/
l
nO. SINTOMATICO
Espasmolfticos
yantiinflamatorios
Reposo e hidratacin
--
Complicado
Obstruccin grave
. Infeccin, febre
Dolor incoercible
Rin nico
Ecografa renal
Dilatacin
INGRESO VITO. AGRESIVO
DRENAJE
(catter o nefrostoma)
no. PARENTERAL
Antibiticos
Remontar hemodinmica
Equilibrio electroltico
Narcticos
VIGILANCIA ESTRECHA
Indican o
no LEO o
ciruga
Tratar la condicin
preexistente
t
CALCICA
acidifcar
(no til si oxalato),
citratos, tiazidas
colestiramina
y dieta baja en
grasas y rica
en calcio, si
hiperoxaluria
RICA
alcalinizar,
alopurinol
ESTRUVITA
acetohidroxmico
CISTlNA
D-penicilamina,
vit. B6 y alcalinizar
Edad?
Tipo de clculos?
Periodicidad de la clnica?
Tipo de slntomas?
Viabilidad renal?
litotricia
t
Ciruga
l
EXTRACORPREA URETEROTOMIA
(LEOCI PIELOLITOTOMIA
PERCUTANEA NERECTOMIA
ultrasonogrfca
con microlumbotoma
ENOOSCPICA
va ureteral
CONTRAINDICADA EN
EMBARAZO
INFECCIN
OBSTRUCCIN DISTAL
Aneurismas
Coagulopatfas
Obesidad
Arritmia cardiaca
Figura 7. Manejo de la litiasis renal ya formada
La presencia de hipertensin
arterial no controlada facilita
el riesgo de hemorragia duran
te l a sesin de litotricia, luego
deber ser estabil izada previa
mente a l a mi sma y constituye,
en cierto modo, por ello, con
traindicacin relativa de LEOC
( MI R 08-09, 93; MI R 99-00F,
1 43) (Tabla 9).
Complicaciones
La expulsin de fragmentos I i
tisicos puede ocasionar un
clico renal y, con menor fre
cuencia, obstruccin ureteral
(steinstrasse o "calle l itisica").
Esta posi bi l i dad es mayor ante
l i ti asi s de gran tamao, por lo
que en al gunos de estos casos
se puede colocar un catter
de derivacin uri nari a (ne
frosloma o doble J) antes de
bies por su pequeo tamao 2-5 mm). Cualquier litiasis podra ser
tratada con LEOC, aunque esto tendr que ser matizado en funcin
la LEOC para di smi nui r este ri esgo, generalmente en l i ti asi s supe
riores a 2 cm.
ABSOLUTAS
RelATIVAS (PRECISAN DE CONTROL PREVIO
Al TRATAMIENTO)
Embarazo
Obstruccin distal
Infeccin activa
Alteraciones de la coaguladn
Aneurisma artico
Alteraciones del ritmo cardiaco, marca pasos
desfbriladores
Obesidad
Hipertensin arterial descontrolada
Tabla 9. Contraindicaciones de la LEO (MIR 9O F, 143)
A un hmbre de 29 a, con antecedentes de dolor tip clico en fosa renal izquierda
q cedi con tratamiento analgico, s l practica una urografa intravenosa, apre.
cinose defecto de replecin radiotransparenle de 6x7 mm e tercio distal d urter
izquierd. El pH de la orina f de 5,5; asimismo, se observan cristales d urato, 9-12
hemates por camp y ea leucocituria. Cul serael tratamiento ms apropiado?
1 ) Alopurinol va oral.
2) Ureleroscopia con exlraccin del clculo.
3) Ncfrolitoloma endoscpica percutnea.
4) Alcalinizlcin de la orina por va oral.
5) Administracin de D-penicilamina.
MIR 0304, 80; RC: 4
Mujer de 50 aos, diabtica insulinodependiente, con infecciones urinarias y clicos
Derivadas del efecto di recto de las ondas de choque, pueden aparecer
contusiones renales manifestadas como hematuria, hematomas rena
les, equimosis O eritema cutneo, y en grado mxi mo, rotura renal. La
hematuria se considera la complicacin ms frecuente de la litotricia.
Ms controvertida es la terica relacin de la LEOC con la aparicin de
hi pertensin arterial, ya que no est demostrada en las ltimas revisiones
publicadas, aunque s la relacin entre hematoma renal post-LEOC e
hi pertensin arterial.
das de evolucin, asociado en las llimas 24 horas a fiebre, escalofros y malestar
general. Analtica de snge: plaquetopenia, leucocitosis y disminucin de la activi
dad de la protrombina. Analtica de orina nrmal. Radiografiade abomen con daras
imgenes de litiasis. Eco renal: dilatacin moderada de sistema excretor izquierdo.
Cul es la conducta ms adecuadal
1 ) Solicitar hemocultivos y urocultivo para cstablecer la necesidad de antibiotc
rapia.
2) Realizar urografa intravenosa para intentar filiar la causa.
3) Hidratar a la paciente bajo observacin rigurosa, y repetir ecografa a las 48
horas.
4) Colocar catter doble J o practicar nefrostomfa percutnca de forma inmediata
con coberura antibitica.
5) lapartoma exploradora pJra objetivar causa, y re;lizar tratamiento antes de que
el cuadro e muy eolucionado.
nefrticos de repticin. Acude a Urgencias por dolor en fos renal izquierda de cinco RC: 4
04.
TUMORES RENALES
Orl?ntacln
MIR
El adenocarcinoma renal es un
tema "de moda en el examen
MIR. Cualquier aspecto de
este lema puede aparecer,
pr es fundamental reconocer
el sindrome de Stauffer,
que ha sido recientemente
introducido y preguntado
varias veces consecutivas.
II Preguntas
MIR 09-1 0, 99
- MIR 06-07, 102
. MIR 0506, 105
. MIR 04-05, 105
- MIR 01-02, 109
- MIR OO-Ol, 1 20
- MIR 99-00, 177
- MI R 99-00F, 144
- MIR 98-99F, 146
Aspectos esenciales
El ms frecuente de los tumores slidos renales es el hipernefroma.
El paciente caracterstico es un varn de mediana edad, obeso y fumador.
la trada clsica consiste en hematuria, dolor y masa en flanco, actualmente, lo ms habitual es que sea
incidentaloma (asintomtico). Si produce sntomas, el ms frecuente es la hematuria.
Hay que sospechar tumor renal ante un varicocele izquierdo, de aparicin sbita y que no cede con el
decbito.
El hipernefroma puede producir multitud de sndromes paraneoplsicos. Esto puede complicar bastante el
diagnstico, de ah el sobrenombre de "tumor del internista".
la elevacin de las transaminasas sin afectacin heptica es tpica del hipernefroma (sndrome de Stauffer).
No se debe confundir un quiste simple con un hi pernefroma. los criterios de quiste simple son: contorno
liso, contenido transnico y refuerzo posterior.
la primera prueba de imagen, ante la sospecha de hipernefroma, sera la ecografa.
El tratamiento fundamental del hipernefroma es la extirpacin quirrgica. la quimioterapia y radioterapia
tienen un papel muy secundario.
4. 1 . Carci noma de cl ul as renal es
(adenocarci noma renal, hi pernefroma)
Es el tumor slido renal ms frecuente (90%) (Figura 8) (MI R 99-00, 1 77). Es un tumor fundamentalmente de l a
edad adulta, con mayor incidencia entre los 40 y 60 aos, con predomi ni o en el varn 2: 1 a excepcin de la
variedad cromfoba, tpi ca de las mujeres.
Entre los factores de riesgo que se han impl icado se encuentran el humo del tabaco, el cadmio y la obesidad.
Existen formas familiares que suelen ser mltiples y bilaterales, como en la enfermedad de Von Hippel-Li ndau y,
en menor medida, la esclerosis tuberosa (MIR 00-01 , 1 20). Se han identificado alteraciones cromosmicas que
implican al cromosoma 3.
RECUERDA
la esclerosis tuberosa y la enferme
dad Von Hippel-lindau se asocian
tambin a otro tumor renal, el an
giomiolipoma.
Presentacin
Asimismo, existe una incidencia aumentada en el rin poliqustico,
en la enfermedad qustica adquirida de la insuficiencia renal crnica
y en los riones malformados, como el rin "en herradura". Procede
de las clulas del tbul o contorneado proximal, y microscpicamente
predomi nan las clulas claras sobre las granulares y fusiformes .
La trada clsica: hematuria, dolor y masa en el flanco ocurre nicamente en el 1 0% de los casos y, cuando se
presenta as, generalmente se trata de una enfermedad avanzada.
1 5

"
"
Manual (TO de Medicina y Ciruga, 8.a edicin
Figura 8. Carcinoma de clulas renales
El 30% presenta metstasis a distancia en el momento del diagnstico,
aunque contrariamente, cada vez son ms los hal l azgos incidentales al
real i zar ecografas abdominales de rutina por otra causa, alcanzando
en algunos estudios ms de la mitad de los casos diagnosticados.
la anomala ms frecuente es la hematuria macroscpica o micros
cpica (60%). Otros hallazgos frecuentes son dolor (40%), prdida de
peso (30%), anemia (40%), masa en flanco (24%), HTA (20%), hiper
calcemia (6%), eritrocitosis (3%). El 20% de los pacientes presentan
como cuadro paraneoplsico alteracin de las enzimas hepticas sin
evidencia de metstasis (sndrome de Stauffer) (MI R 09- 1 0, 99; MI R 06-
07, 1 02; MI R 05-06, 1 05).
Ocasionalmente, el adenocarcinoma renal puede producir hormonas
productoras de sndromes clnicos segn l a sustancia secretada. Entre
stas se encuentran pptidos PTH-like, prostagl andinas, prolactina, re
ni na, gonadotropinas o corticoides. la invasin de la vena renal princi
pal en el lado izquierdo puede ocasionar la aparicin de un varicocele
de forma repentina, que no disminuye en decbito.
RECUERDA
la produccin de pptidos puede aparecer en el hipernefroma, pero es
ms tpica de carcinomas epidermoides (pulmn, esfago, etc.).

Di agnstico
o
E
.
La ecografa es la primera prueba complementaria que debe realizarse,
de forma que la identificacin ecogrfica de una lesin que cumple cri
terios de qui ste simple (contorno liso, contenido transnico y refuerzo
posterior) hace innecesarios mayores esfuerzos diagnsticos, pudiendo
hacerse un seguimiento ecogrfico anual (MIR 99-00F, 1 44). De esta
forma se diagnostican l a mayora de las masas renales en la actualidad
(Figura 9).
1 6
Figura 9. Ecografa de quistes renales simples
la realizacin de puncin-aspiracin con aguja fina (PAAF) de una
masa renal para su filiacin es una exploracin agresiva que, debido
a su baja sensibilidad, no se justifica actualmente, excepto en casos
excepcionales.
la tomografa axial computarizada (e) e el mejor mtodo aislado para
evaluar una masa renal, proporcionando informacin precisa sobre mets
tasis ganglionares (80%) y afectacin de rganos adyacentes (Figura 1 0).
La RM, aunque no se emplea de manera rutinaria en este tipo de pa
cientes, s se utiliza como mtodo de di agnstico bsico en sujetos en
los que se sospecha afectacin lrombtica tumoral de la vena renal o
cava (MIR 04-05, 1 05) (Figura 1 1 ).
Aunque l a urografa intravenosa (UIV) contina siendo l a base del diag
nstico por i magen en urologa, en el caso del adenocarcinoma renal
proporciona pocos datos y de forma i ndi recta, como puede ser la dis
torsin del sistema colector, su ocupacin o l a anulacin funcional del
rin. En las placas tomogrficas de la UIV puede observarse l a presen
ci a de una masa o una alteracin del contorno renal.
Figura 1 1 . RM de tumor renal con trombo en venas renal ' cava
La arteriografa renal, exploracin obligada hace aos, ha quedado re
legada a los casos dudosos, riones nicos y otras situaciones en las
que se pl antea tratamiento qui rrgico conservador. El patrn arteriogr
fico caracterstico incluye neovascularizacin tumoral, lagos venosos,
fstulas arteriovenosas y vasos capsulares.
El estudio de extensin, si se sospechan metstasis, se completar rea
lizando radiografa de trax, analtica heptica completa y, en algunos
casos dudosos, gammagrafa sea.
El procedimiento diagnstico ante la presencia de masas renales se
puede observar en la Figura 1 2.
MASA RENAL descubierta accidentalmente
. Examen fsico
.
Anlisis de orina
Quiste complejo
o masa slida
Masa slida
Quiste simple
Observar
o quiste complicado
Angiomiolipoma
NEFRECTOMIA
RADICAL
O PARCIAL

No complicado
OBSERVACiN
Complicado
Nefrectoma simple
Nefrectoma parcial
Embolizacin
Figura 12. Algoritmo diagnstico de las masas renales

rOIOg a
Tratami ento
Una vez estudiado el tumor y descartada la presencia de metstasis,
tanto viscerales como ganglionares, el tratamiento de eleccin es la
nefrectoma radical, incl uyendo la fascia de Gerota y la glndula su
prarrenal (MIR 01 -02, 1 09).
En ocasiones especiales se puede plantear l a ci ruga conservadora o
parcial, como en los tumores bi laterales, en aquellos que aparecen so
bre rin nico, o sujetos con nefropatas mdicas, en los que la prdi
da de masa nefronal obligara a di l isi s.
Adems de en estas i ndicaciones imperativas, actualmente es el nuevo
patrn de referencia en el tratamiento de tumores pequeos (menos de
4 cm), bien delimitados y si n afectacin de la grasa perirrenal. En estos
pacientes seleccionados parece que la supervivencia y la tasa de reci
divas locales son semejantes a las que se presentan en casos si mi l ares
tratados con nefrectoma radical.
La l i nfadenectoma regional no mejora l a supervivencia y nicamente
tiene, por tanto, validez en la estadificacin, por lo que no se realiza
sistemticamente.

ste se establece sobre la base de los hal lazgos qui


rrgicos y anatomopatolgicos.
Otras formas de tratamiento carecen de eficacia. Tanto la qui miotera
pia como la radioterapia ofrecen resultados pobres. En el caso de en
fermedad metastsica, las opciones son mlti ples, pero ninguna satis
factoria. Aunque se ha descrito la regresin de las lesiones metastsicas
tras la nefrectoma, esto ocurre nicamente en un 1 %, Y generalmente
de forma transitoria, por l o que no se justifica, salvo de forma paliativa
por otros motivos.
La i nmunoterapia con i nterferones, interleucina, l i nfocitos killer acti
vados, y ciertas combinaciones de qui mioterapia con i nmunoterapia,
son al ternativas para l a enfermedad metastsica, pero en ni nguna de
ellas se obtienen tasas de respuesta superiores al 1 5%. Actualmente
se uti l i zan de preferencia: antitirosincinasas en primera lnea, an
tiangiognicos en segunda lnea. Las situaciones que favorecen l a
respuesta al tratamiento i nmunoteraputico son: presenci a de me
tstasis pul monares exclusivamente, buen estado general, y que se
haya realizado la nefrectoma antes del descubrimiento de las masas
pul monares .
En la actualidad se i nvestiga sobre autovacunas elaboradas con l i nfoci
tos peritumorales que parecen ofrecer resultados alentadores.
4. 2. Otros tumores
Tumor de Wilms (vase Seccin de Pediatra) .
Tumores renales metastsicos. Pueden encontrarse metstasis en el
rin de tumores de pulmn (la ms frecuente), mama, mela nomas
e i nfi ltracin por l i nfoma.
Tumores benignos:
Adenomas corticales: son los tumores ms frecuentes del adulto,
aunque indistinguibles clnicamente del adenocarcinoma, por lo
que se tratan como tales. El criterio clsico de tamao (3 cm)
para su diagnstico diferencial no es vlido en l a actualidad.
Angiomiolipomas: se asocian a l a esclerosis tuberosa en un 50%
(MI R 98-99F, 146). Compuestos de una proporcin variable de
grasa, vasos y fibras musculares. Cuando son grandes (mayores
1 7
Manual ero de Medicina y Ciruga, B.a edicin
1 8
de 4 cm), pueden ocasionar u n sndrome de Wnderlich por
sangrado relroperitoneal . Cuando se asocian a esclerosis tubero
sa, suelen ser mltiples y bi laterales, por lo que deben tratarse de
forma conservadora.
Oncocitoma: considerado benigno, aunque en algunos se han
detectado metstasis. Hay criterios radiolgicos para dislin-
Un hombre de45 aos tiene un carcinoma de clulas renales extendido. los niveles
de GOl, fosfatasa alcalina, lOH y a-2 globulina son elevados y el tiempo de protom
bina alargado. El hgado aparece difusamente agrandado, pero no existen defectos
focales de infiltracin intraheptica. l explicacin etiolgica ms probable para
estos hallazgos ser:
1 ) los efectos hcpdlolxicos de tumor.
2) Metsl,Sis hep.ltica.
3) Amiloidosis.
4) Trombsis tumorales que obstruyen la vena heptica.
5) Hepatitis vrica aguda.
MIR 05-06, 105; RC: 1
gui rl o del adenocarci noma, pero en la mayora de los casos,
ni stos ni la ci tologa o la biopsia, ofrecen garantas sufi
ci entes de su beni gni dad, por 10 que tienden a ser tratados
mediante nefrectoma.
Nefroma mesoblstico (hamarloma fetal): es el tumor benigno
ms frecuente en recin nacidos y lactantes.
Casos clnicos representativos
Un paciente de 62 aos, con alteracin de la funcin renal y crisis de hematuria,
presenta una masa abdominal palpable en nanco derecho. Se le realiza una le, de
tectndose una masa de carcter slido de 8 cm de dimetro en rin derecho. En la
anamnesis destaca que el paciente es fumador de 35 cigarrillos al da. Cul es, entre
los siguientes, el diagnstico de presuncin ms probable?
1 ) Nefroblasloma.
2) Upsarcoma.
3) Angiomiolipoma.
4) Adnocarcinoma.
5) Carcinoma epidermoide.
MIR 99-0 , 1 7; RC: 4
05.
HI PERPLASIA ` CARCINOMA
PROSTTICO
OrientacIn
MIR
Tanto la hiperplasia
prosttica benigna como
el cncer de prstata, son
dos temas fundamentales.
Probablemente el cncer sea
ms importante, sobre todo
en lo referente al tratamiento.
Hay que aprenderse muy bien
el resumen de la Tabla 1 1;
aporta muchas preguntas
acertadas a cambio de poco
esfuerzo.
i Preguntas
MIR 08-09, 106
- MIR 06-07, 1 03, 233
- MIR 05-06, 106
- MIR 04-05, 1 06, 225
- MIR 03-04, 91
- MIR 02-03, 188
- MIR 01-02, 1 04, l OS, 1 07
- MIR aQ-Q1 , 1 1 9
- MIR 98-99, 1 37
- MI R 98-99F, 147
[
Aspectos esenciales
La hiperplasia prosttica benigna (HPB) suele afectar a la zona periuretral de la glndula. El cncer aparece
en la zona perifrica.
La hiperplasia prosttica benigna no guarda relacin con el cncer.
Tanto l a HPB como el cncer tienen relacin con las hormonas sexuales, y suelen aparecer en varones
ancianos.
El tratamiento mdico de la HPB consiste en a-bloqueantes (relajan l a musculatura uretral y del cuello
vesical), inhbidores de l a 5 a-reductasa (disminuye el tamao glandular) y (toterapa. Esta ltima no ha
demostrado utilidad con parmetros objetivos.
El tratamiento definitivo de la HPB es la ciruga, que puede consistir en reseccin transuretral o en ciruga
abierta, dependiendo del tamao prosttico.
El cncer de prstata es casi siempre un adenocarcinoma, con gran frecuencia multifocal.
El cncer de prstata cada vez se diagnostica con ms frecuencia en fase asintomtica. Cuando presenta
clnica, puede consistir en sntomas urinarios simi lares a la HPB.
El PSA elevado no es diagnstico de cncer de prstata. Puede corresponder a una HPB. El diagnstico defi
nitivo de cncer prosttico precisa una biopsia.
Las metstasis lumbares son tpicas del cncer de prstata, pudiendo producir compresin medular.
El tacto rectal revela una prstata ptrea e irregular en el cncer de prstata. Sin embargo, al principio puede
no ser palpable, ni visible en la ecografa (T1).
La principal complicacin quirrgica del cncer de prstata es l a impotencia.
Ante un sndrome de compresin medular por cncer de prstata, nunca se deben emplear anlogos de la
LHRH nicamente. Siempre deben asociarse antiandrgenos.
En el cncer de prstata, la indicacin ms clara de prostatectoma radical es el estadio T2a.
El tratamiento fundamental del cncer de prstata diseminado es la hormonoterapia.
5. 1 . Hi perpl asi a prosttica beni gna
La hiperplasia prosttica benigna (HPB) afecta en mayor o menor grado a la gran mayora de los varones a partir
de la qui nta dcada de l a vida, alcanzando el 80-95% de l a poblacin mascul i na de 80 aos.
La prstata se divide clsicamente en cinco lbulos (anterior, medio, posterior y dos laterales); aunque stos ni
camente se encuentran como tales en la edad fetal . En el adulto se puede i nterpretar la anatoma de l a prstata
dividida en dos partes: una zona perifrica, donde se origina principal mente el carcinoma, y una zona periure
tral o transicional, de la que procede la HPB (Figura 1 3) .
La HPB est compuesta de una proliferacin variable de elementos glandulares, musculares y del estroma, que en
su crecimiento comprimen la prstata perifrica, formando la l l amada cpsula quirrgica. Su etiopatogenia no est
clara; aunque el estmulo andrognico a travs de su forma activa, la di hi drotestosterona, es fundamental, su papel
exacto no ha sido determinado. Las teoras ms recientes abogan por un desequi l ibrio hormonal de estrgenos/an
drgenos, o fr la existencia de factores de crecimiento prostticos con un papel permisivo del ambiente hormonal.
No existe evidencia de asociacin entre HPB y carcinoma prosttico.
1 9
8
"
1
e
"

E
o
e
'2

u
>

.
,

c
"
"-
I
Manual eTO de Medicina y Ciruga, B.a edicin
Lbulo
lateral
Estroma fbromuscular anterior
Lbulo anterior
\?7
-Zona perifrica
zona central
Glndulas
suburetrales
Conductos
eyaculadores
Lbulo posterior
Figura 13
.
Anatomra de la prstata
Di agnstico
El crecimiento prosttico generalmente se produce hacia l a uretra,
ocasionando obstruccin de sta y dificultando el vaciamiento vesical
(Figura 1 4). Esto no se manifiesta inmediatamente, sino que, general
mente, el proceso pasa por una serie de etapas que incluyen una fase
de compensacin, una fase clnica y una de descompensacin.
20
Figura 14. Ecografa de hiperplasia prosttica con crecimiento
del 16bulo medio intravesical
Fase de compensacin. El crecimiento prosttico ocasiona un au
mento de la presin uretral durante el vaciado que es compensado
por una mayor actividad contrctil del detrusor que se hipertrofia,
encontrando presiones vesicales ms elevadas. En esta fase, la clni
ca puede ser mnima o inexistente.
Fase clnica (Figura 1 5). La elongacin de las fibras musculares por
encima de un lmite condiciona prdida de capacidad contrctil. En
este momento aparece retraso del inicio de la miccin, disminucin
del calibre y de l a fuerza del chorro micdonal y alargamiento del
vaciado (10 que en conjunto se denomina sndrome prosttico), El va
ciado suele ser incompleto, dando lugar a un residuo postmiccional.
Urografa lntraveno mostrando Impronta prosttica en vejiga (vejiga en montera")
con urteres en anzuelo
Figura 1 S. Hiperplasia prosttica en fase clnica
Fase de descompensacin (Figura 1 6). Se produce un vencimiento
del detrusor vesical, que es i ncapaz de vencer la presin uretral, au
mentando la sintomatologa anterior y pudiendo presentarse reten
cin urinaria. Ocasionalmente puede aparecer di latacin ureteral
bi lateral con deterioro de l a funcin renal. Esto se debe a uropata
obstructiva i nfravesical con prdida del mecanismo anti rreflujo.
Situacin del mismo paciente un ao despus de la anterior: hidronefrosis grave.
De la vejiga (no seobserva) se evacuaron 3500ce de orina
Figura 16. Hiperplasia prosttica en fase de descompensaci6n
Pueden producirse tambin otro tipo de sntomas denominados "i rrita
tivos", que son debidos a la alteracin funcional vesical, y cuya reso
lucin es ms difcil tras la desaparicin de la obstruccin (MIR 01 -02,
1 04). Entre estos sntomas se incl uyen polaquiuria, tenesmo, nicturia y
urgencia mlccional. La HPB es la causa ms frecuente de obstruccin
del tracto urinario inferior en el varn.
En la evaluacin del sndrome prosttico, el tacto rectal contina sien
do l a exploracin fundamental, sobre todo para diferenciarlo del car
cinoma, ya que no es i nfrecuente que ambas entidades coexistan. La
cl nica es lo ms i mportante para valorar la indicacin de tratamiento
de la HPB, ya que no existe correlacin entre el tamao prosttico y el
grado de obstruccin ( MI R 08-09, 1 06). Cualquier zona sospechosa al
tacto debe ser biopsiada.
La medicin del flujo mximo miccional es tambin importante, conside
rndose normal cuando es mayor de 1 5 mi/s y claramente patolgico si
es menor de 1 0 mi/s. El estudio puede completarse con una ecografa que
permita evaluar si existe afectacin del tracto urinario superior, residuo
postmiccional, l i tiasis vesical u otra patologa asociada. El uso del PSA en
la HPB nicamente est i ndicado para descartar l a presencia de carcino
ma en l a prstata, ya que no sirve para diagnosticar HPB, aunque recien
temente ha demostrado ser el mejor predictor de la historia natural de la
enfermedad. Es decir, que mayores niveles de PSA en HPB diagnosticada
probablemente se correlacionarn con mayores volmenes prostticos y
con ms posibilidades de complicacin derivadas de la HPB.
Tratami ento
Dentro de las posibilidades teraputicas, la ci ruga contina siendo el
nico tratamiento definitivo para la HPB.
t
sta puede ser endoscpica
(RTUp: reseccin transuretral prosttica) o abierta (adenomectoma pros
ttica) (Figura 1 7), dependiendo del tamao del adenoma. En el 1 0% de
las piezas obtenidas se encontrarn focos de adenocarcinoma incidental.
Figura 17. Adenomectomia prosttica
Se debe tener en cuenta que en la ciruga de la HPB no se extirpa la
cpsula quirrgica, que est constituida por las glndulas prostticas
perifricas comprimidas por el adenoma, y es el principal origen del
carcinoma prosttico, por lo que la i ntervencin quirrgica no protege
del desarrollo de este proceso.
Los tratamientos no qui rrgicos incluyen una variedad de fitoterapias,
poco efectivas si se valoran con parmetros objetivos, inhibidores de la
5 a-reductasa (finasterida, dutasterida) (MIR 04-05, 225) que reducen el
tamao prosttico, antagonistas a-adrenrgicos (alfuzosina, prazosn,
urolo9ia _
doxazosina, terazosina, tamsulosina, etc.) que relajan la musculatura
del cuello vesical y uretra. Hasta ahora, estos frmacos se han estado
utilizando en forma de escalera teraputica, pero la aparicin del es
tudio COMBAT parece indicar que en pacientes con si ntomatologa a
partir de moderada, y con volmenes prostticos por encima de 30-40
ce se debe realizar terapia combinada de inicio.
RECUERDA
La finasterida tambin es til para l a alopecia andrognica, donde se
emplea en dosis mucho menores.
Como inconvenientes principales de los i nhi bi dores de la 5 a-reductasa,
se encuentran: i mpotencia, reduccin del PSA en torno al 50% (difi
cultando el diagnstico del carcinoma, si lo hubiese) y que tarda una
media de cuatro meses en hacer efecto.
De los a-bloqueantes, el inconveniente principal es la hipotensin.
En cuanto a las indicaciones de tratamiento qui rrgico, globalmente,
slo un 1 0% de los pacientes prostticos precisar ciruga. La inten
sidad de las manifestaciones cl nicas subjetivas y la mala respuesta al
tratamiento mdico pueden constituir la indicacin para la i nterven
cin. Entre las causas "objetivas" que suponen indicacin absoluta de
tratamiento quirrgico se encuentran (MIR 01 -02, 1 05):
Retencin urinaria reiterada.
Hidronefrosis retrgrada (lesin del parnquima renal por obstruc
cin infravesical).
Infeccin urinaria de repeticin.
Litiasis vesical.
Hematuria de repeticin.
5. 2. Carci noma prosttico
El adenocarcinoma prosttico es el tumor maligno ms frecuente del
aparato genitourinario mascul i no y el segundo en frecuencia general,
despus del pulmonar. Sin embargo, si se incl uyesen los carcinomas
incidentales y los encontrados en autopsia, supera al pulmonar en pre
valencia (MIR 06-07, 1 03).
La hormonodependencia del cncer prosttico parece indicar el papel
de los andrgenos en su etiologa o patogeni a. La relacin de factores
genticos, ambientales o i nfecciosos no ha quedado suficientemente
establecida.
El 95% de los carcinomas prostticos son adenocarcinomas origi nados
en la zona perifrica de la prstata. Los carcinomas ductales se originan
en los conductos prostticos en lugar de los acinos, e hi stolgicamente
pueden corresponder a carcinomas transicionales, escamosos, endome
trioides o mixtos. Ms raros son los carcinosarcomas (menos del 1 %).
El adenocarcinoma prosttico, con frecuencia, es multifocal y presenta
poblaciones en distinto grado de diferenciacin. En esta heterogenei
dad se basa la clasificacin de Gleason, que asigna una puntuacin de
1 a 5, segn el patrn histolgico de cada una de las dos poblaciones
ms representativas de la masa, sumando ambas puntuaciones para
obtener un resultado final de 2 a 1 0. Esta escala de Gleason se corres
ponde con el pronstico de la enfermedad, independientemente del
estadio (MI R 06-07, 233). Para la estadificacin se emplea principal
mente la clasificacin TNM (Tabla 1 0 y Figura 1 8) (MIR 98-99F, 1 47).
21
Manual eTO de Medicina y Ciru
ga, s.a edicin
. T1: tumor naparente cI(nicamente (no palpable ni visible por tcnicas de
i magen)
T1 a: hallado incidentalmente. Afctacin menor del 5% del tejido resecado
Tl b: hallado incidentalmente. Afectacin mayor del 5% del tejido resecado
Tlc: Tumor identifcado por puncin-biopsia por aumento del PSA
T2: tumor confnado a la prstata (incluye la invasin de la cpsula prosttica
sin exteriorizacin del tumor hacia el tejido adiposo periprosttico)
T2a: menos del 50% de un lbulo
- T2b: ms del 50% de un lbulo
- T2c: dos lbulos
T3: extensin del tumor por fuera de la cpsula
- T3a: extensin transcapsular (sea unilateral o bilateral)
- T3b: invasin de la(s) veslcula(s) seminal{es)
T4: tumor fjo o invade rganos adyacentes distintos a las vesculas
seminales (cuello vesical, esfnter externo, recto, msculo elevador
o pared
NX: no se pueden estudiar los ganglios regionales
NO: no metstasis ganglionares
N 1: metstasis a
MO: no metstasis
Ml : metstasis a distancia
Ml a: ganglios linfticos no regionales
Mlb: hueso
M 1 c: otras local izaciones
Tabla 1 0. Estadifcacin del carcinoma de prstata
Cl nica
El carcinoma prosttico es una enfermedad ms frecuente en ancia
nos, y la mayora de ellos se di agnostica por encima de los 60 aos .
Cl ni camente puede producir sntomas obstructivos del tracto urina
rio i nferior superponibles a l os de l a HPB. A stos puede aadi rse l a
hematuria. El 25% de los pacienles que refieren retencin uri nari a
aguda presentan un carcinoma prosttico. Aproximadamente un 25%
de l os pacientes presentan metstasis en el momento del diagnstico;
stas pueden producir manifestaciones como dolor seo, compresin
medular, mieloptisis o coagulopata. Afortunadamente, estos casos
se encuentran en claro descenso gracias a l a i ncorporacin del PSA
(prostate-specic antigen-antgeno prosttico especfico), facilitando
el diagnstico de l a enfermedad en estadios tempranos y comnmen
te asi ntomticos.
Di agnstico
Tacto rectal
Contina siendo el mtodo fundamental de cribado. Son accesibles al
tacto rectal todos los estadios excepto el T1, que por defi nicin es un
hallazgo. Caractersticamente, el carcinoma es duro, nodular e irre
T: tmor primario cUnlca TN
na
22
TX NO puede evaluar el tumor
TO No existen signos de tumor primario
na nb
<5% >5%
T1 Tumor no evidente dlnicamente, no palpable
ni visible mediante tcnicas de imagen:
na Extensin menor o igual a1 5% del tejido
resecado
. Tl b Extensin mayor del 5% del tejido
resecado
. TlcTumor identifcado mediante puncin
bipsica (consecuencia de un PSA elevado)
Tla Tlb
Tlc
T2 Tumor limitado a la prstata o a la
capsula, sin sobrepasarla:
T2a Menos del 50% de un lbulo
T2b Ms del 50% de un lbulo
T2c Dos lbulos
n
nb
T Tumor que se extiende a travs de la
capsula prosttica:
. TIa Extensin extracapsular (unilateral
o bilateral)
. TIb Tumor invade la veslcula seminal
N: ganglios linfticos regionales
T4Tumor fjo o que invade estructuras adyacentes diferentes a las veslculas seminales NX No se pueden evaluar los ganglios linfticos regionales
NO No hay metstasis ganglionares regionales
Nl Metstasis en ganglios linfticos regionales
Figura 18. Estadifcacin del adenocardnoma de prstata
guiar. En general, se aconseja un tacto rectal y un PSA anual a todos
los varones por encima de 50 aos aunque, de momento, la OMS no
aconseja l a realizacin de cribado poblacional si stemtico.
Marcadores tumorales
Se dispone fundamental mente de dos marcadores tumorales. La fosfa
tasa cida prosttica (FAP) se emplea en clnica desde hace dcadas,
es un marcador especfico, pero su elevacin suele indicar extensin
extraprosttica, por lo que no resulta ti l en el di agnstico precoz. El
antgeno prosttico especfico (PSA) es realmente un marcador de te
ji do prosttico cuyos ni veles suelen encontrarse ms elevados en el
cncer, pero es i nespecfico y tambin estn elevados a consecuencia
de patologa benigna (i nfecciones, sondajes, HPB, etc.). Por este mo
I) LKcorte transversal; {b} E corte longitudinal; (el Adenocarclnoma,
ndulo hlpoecojco en lbulo derecho
Figura 19. Ecografra transrectal (ETR) de adenocarcinoma prosttico
Ur01 09a _
tivo se ha i ntentado aumentar su especificidad para cncer con otros
parmetros (densidad de PSA, ndice PSNedad, velocidad de cambio
del PSA, PSA libre), aunque an no ha quedado establecida l a ventaja
de stos sobre el PSA ai slado.
Si el PSA es menor de 4 ng/ml, es poco probable que se encuentre un
cncer de prstata. Si es mayor de 1 0, las probabi l idades aumentan, lo
que aconsejara una biopsia de prstata ecodi ri gi da. Si est entre 4 y
1 0, se pueden uti l i zar los parmetros antes mencionados para valorar
la necesidad de biopsia (MIR 01 -02, 1 07).
Pruebas de imagen
La ecograra transrectal (ETR) (Fi gura 1 9) es el mtodo de imagen ms
til para l a esladificacin local, pudiendo ofrecer informacin i mpor
tante sobre la afectaci n capsular, de vesculas seminales, cuello vesi
cal o recto. Aunque no existe un patrn caracterstico, suele aparecer
como ndulos hipoecognicos. La ecografa transrectal ofrece, ade
ms, la posibilidad de diri gi r la biopsia hacia las zonas sospechosas.
La ecografa abdominal no tiene gran valor en la deteccin del car
ci noma prosttico. La TC y la RM tienen su principal papel en la es
tadificacin ganglionar y la valoracin de metstasis a distancia. Las
primeras metstasis deben buscarse a nivel de los ganglios l i nfticos de
las cadenas obturatrices e i l acas.
Gammagrafa sea
Se utili za para la deteccin de metstasis seas, tiene mayor sensibi
lidad que l a radiologa convencional (Figura 20), y debe reali zarse en
todo paciente en quien se sospeche metstasis (Gleason > 8, PSA > 20).
Antes de plantearse el tratamiento curativo, en ciertos pacientes con al
tas probabi l idades de encontrarse el cncer extendido, se debe efectuar
una gammagrafa previa para confirmar l a no existencia de metstasis
seas o una TC para descartar metstasis ganglionares.
Figura 20. Radiografa de columna. Metstasis osteoblsticas
23
Manual CTO de Medicina y Ciruga, 8," edicin
RECUERDA
las metstasis del cncer de prstata son osteoblsticas, es decir, for
man hueso (no en la gammagrafa, sino en la radiografa).
Biopsia prosttica
Debe reali zarse para la confirmacin del di agnstico. Puede efectuarse
va transrectal o transperineal, guiada por el tacto rectal o bien guiada
por l a ecografa transrectal, lo que aade efectividad a la prueba. la
realizacin de la biopsia est i ndicada siempre que exista una anoma
la del tacto rectal, elevacin de los marcadores tumorales o al teracin
en las pruebas de imagen. la puncin-aspiracin con aguja fina (PAAF)
es una alternativa con menores complicaciones, pero con el i nconve
niente de que no puede evaluar el grado histolgico (Gleason).
RECUERDA
Son indicaciones de biopsia prosttica el tacto rectal sospechoso, la
presencia de un ndulo ecogrfico y un PSA > 4 (variable la cifra segn
criterios).
Tratami ento
Opciones teraputicas
24
Prostatectoma radical. los pacientes candidatos deben ser individuos
con una esperanza de vida superior a 1 0 aos. Como complicaciones,
se puede encontrar incontinencia (2-57%), estenosis anastomtica
( 1 0%), impotencia (50%) o incluso la muerte 5%). En l neas gene
rales, suele ir acompaada de l i nfadenectoma leo-obturatriz.
Radioterapia. Como tratamiento curativo I los resultados en estadios
localizados se acercan a los de l a ci ruga. la di arrea crnica, la
proctitis, la cistitis rdica y las fstulas urinarias son complicaciones
del tratamiento, as como l a incontinencia y la i mpotencia a partir
de los dos aos de tratamiento.
Se ha empleado tambin radioterapia intersticial (braquiterapia) con
implantacin de yodo-1 23 (1-1 23), oro-1 98 (Au-1 98), paladio e iri
dio. Su indicacin queda l i mitada a tumores pequeos de estadio T1
o T2, y sus resultados son si mi lares a los de l a ci ruga. En caso de
compresin medular o dolor por metstasis seas, l a radioterapia so
bre la metstasis puede consegui r el control local de la enfermedad.
Hormonoterapia. El adenocarcinoma prosttico est compuesto por
una poblacin heterognea de clulas andrgeno-dependientes y
andrgeno-independientes. La supresin hormonal frena el creci
miento de las primeras, pero no afecta a las andrgeno-indepen
dientes. Se puede consegui r di smi nui r los ni veles de andrgenos
circulares por disti ntos mtodos:
Castracin quirrgica. Es el mtodo ai slado ms eficiente, con la
ventaja de que el i mi na la necesidad de medicacin permanente.
Por su rapidez en el efecto supresor hormonal, tambin est in
dicada en las compresiones medulares por metstasis.
Estrgenos (dietilestilbestrol). I nhi be l a secrecin de LH. Actua
lemente este mtodo se ha abandonado debido al alto riesgo
cardiovascular que conlleva.
Progestgenos. Inhiben l a secrecin de lH y actan como an
tiandrgenos, al uni rse a los receptores de l a di hi drotestosterona.
Es preciso aadir estrgenos para evitar el fenmeno de escape,
que se produce tras varios meses de tratamiento. No son de uso
habitual.
Agonistas LHRH. Aunque i ni ci al mente ocasionan un aumento
de los ni veles de testosterona, posteriormente suprimen l a se
crecin de lH y de andrgenos. la elevacin transitoria de los
andrgenos puede empeorar el cuadro clnico, principal mente
si existe compromiso medular por metstasis seas. Esta eleva
cin rflare-up) se debe suprimir mediante l a admini stracin de
antiandrgenos, previamente a la introduccin de i nhi bidor de
la LHRH (MIR 00-01 , 1 1 9).
Antiandrgenos (bicalutamida, flutamida, acetato de ciprotero.
na). Compiten con el receptor andrognico. Suelen utilizarse con
con agonistas de la lHRH. El acetato de ciproterona, adems de
actuar como antiandrgeno, tiene un efecto progestgeno, por lo
que acta a nivel central, di smi nuyendo los pulsos de lH.
Quimioterapia. No es muy efectiva en el adenocarcinoma pros
ttico. Se han realizado tratamientos con frmacos, que son una
mezcla de un estrgeno y una mostaza nitrogenada (fosfato de
estramustina), pero los estudios son contradictorios y las res
puestas pobres. Existen estudios prometedores con el empleo de
docetaxel en pacientes con tumores hormonorresistentes.
Tratamiento por estadios (MIR 04-05, 1 06)
Estadio Tla. Tienen una mortalidad por la enfermedad del 2% a los
1 0 aos, por lo que no precisan tratamiento, salvo quizs los pacien
tes jvenes (menores de 60 aos) con una elevada esperanza de vida.
Estadio Tlb-Tlc. Alcanzan una mortalidad del 80% dejados a su
evolucin natural. Por ello est indicada la prostatectoma radical,
l a radioterapia externa o braquiterapia, en sujetos con esperanza de
vida superior a 1 0 aos (MI R 02-03, 1 88) .
Estadio T2a. Es la indicacin ms clara de prostatectoma radical. la
radioterapia o braquiterapia se reservara para pacientes de riesgo
qui rrgico elevado o que no aceptan efectos secundarios atribuibles
a l a ciruga.
Estadio T2b y T2c. Un 40% demuestra ser en realidad estadio 3, tras
el anlisis de la pieza quirrgica de prostatectoma radical ( i nfraes
tadificacin). la radioterapia externa o braquiterapia tambin puede
ser ti l en pacientes de alto riesgo qui rrgico.
Estadio T3a. la i ndicacin quirrgica es dudosa, as como la radio
terapia local, por lo que solamente se propondra a sujetos jvenes,
aun a costa de obtener malos resultados. Generalmente son tratados
como el grupo siguiente.
Estadio T3b, T4, N+, M+. Varn aoso con mal estado general. El
tratamiento hormonal es la opcin indicada. Puede ser preciso el
uso de radioterapia paliativa sobre la metstasis en caso de dolor
(MIR 03-04, 91 ; MI R 98-99, 1 3 7).
Recidiva tumoral posterior a tratamiento con intencin curativa
Tras la realizacin de prostatectoma radical, los pacientes son monito
rizados generalmente con peticiones de PSA
Cuando las cifras de PSA tras prostatectoma radical son superiores a
0,4 ng/ml, se considera recidiva bioqumica y debe hacer sospechar l a
existencia de metstasis a distancia, o bi en la existencia de recidiva a
nivel local (MI R 05-06, 1 06)_
Tras l a realizacin de radioterapia como tratamiento de cncer de
prstata localizado, los descensos de PSA van siendo paulatinos (a di
ferencia de la prostatectoma radical) hasta conseguir un valor nadir,
que es el valor mnimo alcanzado tras el tratamiento y que se con
siderar referencia para el seguimiento posterior. Existen diferentes
criterios para considerar el diagnstico de recidiva bioqumica tras
tratamiento con radioterapia: cuando se constata l a existencia de tres
elevaciones sucesivas a partir del valor nadir, cuando se evidencian
ni veles nadir+2 (criterio de la ASCO), o con valores nadir+3 (criterio
de Phi l adelphial.
Tratamiento de urgencia
La compresin medular por el cncer prosttico no tratado puede ser la
forma de presentacin y constituye una urgencia importante. El objetivo
del tratamiento debe ser la supresin andrognica rpida o la descom
presin medular mediante lami nectoma quirrgica o radioterapia. Se
pueden disminuir los niveles de andrgenos mediante castracin quirr
gica urgente, ketoconazol en altas dosis o dieti lesti l bestrol i ntravenoso.
A continuacin se expone una tabla que resume tanto las caractersti
cas de la hi perplasia prosttica benigna (HPB) como las del adenocar
cinoma prosttico (Tabla 1 1 l.
Paciente de 66 aos, intervenido de prostatecloma radical, hace 3 aos por ade
nocarcinoma de prstata Cleason 8 (pT2b NOMO). Presenta, en el momento actual,
una cifra de PSA srico de 12 nglml. Seale cul de las siguientes afirmaciones le
parece correcta:
1 ) La supervivencia media en el momento actual es menor de 1 ao.
2) La cifra de PSA est en rango normal ya que existen otras fuentes de produccin
del mismo.
3) El paciente puede tener una recidiva local o bien melstasis a distancia.
4) La utilizacin de bloqueo hormonal en este caso no es una opcin de tratamiento
posible.
5) En caso de tratarse de una recidiva local, estara indicado realizar ciruga de res.
cate para extirpar dicha masa.
MIR 05-06, 106; RC: 3
Hombre de 77 aos que refiere clnica de prostatismo de aos de evolucin, y que
presenta elevacin del PSA (antgeno prosttico especifico) (89 ng/ml) y dolor en
columna lumbar desde hace 2 meses. Al tado rectal, la prstata est aumentada de
tamao, de consistencia dura en ambos lbulos, superficie nodular y lmites mal defi
nidos. Tras realizarle una ecografa transrectal con biopsias prostticas ecodirigidas,
es diagnosticado de un adenocarcinoma de prstata pobremente diferenciado, que
afecta a ambos lbulos y que infiltra las vesculas seminales. La gammagrafa sea
confirma la presencia de metslasis en columna lumbar. Qu tratamiento de los
siguientes aconsejara en primer lugad
1 ) Prostalectoma radical.
2) Quimioterapia intensiva.
3) Hormonoterapia.
4) Radioterapia pelviana externa.
5) Braquiterapia prosttica.
MIR 03-04, 91; RC: 3
UrO,09a _
I
"PB
ADENOCARCINOMA
PROSTATICO
Ltl Zona translcional Perifrica
Fases: 25% slndrome prosttico
Olnlca
Compensacin 25% retencin aguda
CHnica 25% metstasis
Descompensacln
Tacto rectal
Ecografla transrectal (estadifcacin local)
OIognOstlco
Gammagrafla osea (metstasis seas)
d_.1
PSA (muy sensible, poco especifco). Descarta cncer prosttico,
pero no diagnostica HPB
Fosfatasa cida (muy especifca, poco sensible)
Biopsia (confrmacin)
Fitoterapia Localizado: prostatectomla radical
Frmacos: fnasterida, ms Ilnfadenectomla bilateral.
Tratamiento n-bloqueantes radioterapia
Clrugla: adenomectomla: Avanzado: castracin: quirrgica
endoscplca o abierta (eleccin), farmacolgica
Tabla 1 1 . Tabla-resumen de las caracterfstlcas de la HPB
y del adenocarcJnoma prosttico
Casos clnicos representativos
Un paciente de 67 aos acude a Urgencias por presentar en los ltimos das de
bilidad progresiva de miembros inferiores, dificultad miccional e incontinencia
fecal. En la exploracin fsica destaca cierta hipotona anal, con una prstata muy
sugerente de malignidad al lacto, y debilidad de extremidades, conservando la sen
sibilidad tctil. Con el probable diagnstico de carcinoma de prstata melastsico,
cul de las siguientes opciones considera MENOS indicada para el tratamiento
de urgencia?
1 ) Estrgenos intravenosos.
2) Anlogos LHRH.
3) Ketoconazol (altas dosis).
4) Radioterapia.
5) Orquiectoma bilateral.
MIR 00-01 , 1 1 9; RC: 2
A la consulta acude un paciente de 54 aos con molestias a la miccin. Refiere
disminucin del c,orro, dificultad para el inicio, goteo postmiccional, sensacin de
tenesmo y nicturia de tres veces. Presenta cultivos negativos y PSA de 2,1. En la
ecograffa abdominal se objetiva una glndula prosttica de 43 cc. Al tacto reclal
no se plpan ndulos sospechosos. En el IPSS obtiene una puntuacin que permite
clasificar su sintomatologa de moderada-grave. Su actitud deber ser:
1) Debido a su edad, el primer paso ser iniciar tratamiento con Htoterapia.
2) Debido a la gravedad de los sntomas s debe plantear ciruga de entrada.
3) Se d iniciar tratamiento con a-bloqueantes.
4) L mejor opcin seri nicia r trata miento combi nado con a-bloqueantes+ inhibidores
de la 5 a-reductasa.
5) Se debe iniciar tratamiento con inhibidores d la 5 a-recuctasa.
Re: 4
25
Onentacln
MIR
Las preguntas sobre este
tema suelen ser senciHas
y repetitivas, aunque
ltimamente ha aparecido
como nuevo concepto el
carcinoma in silu. El estudio
del Desglose es especialmente
importante, pero se debe
tener en cuenta que la
tendencia parece orientada
a preuntarse cada vez ms.
Es fundamental la parte de
tratamiento.
e Preguntas
- MIR 07-8, 102
- MI R 06-07, 96
- MIR 05-06, 103
- MIR 04-05, 259
- MIR 03-04, 82
- MI R 02-03, 1 79
- MIR 00-01, 1 1 8
- MI R oo-OI F, 1 43, 144
- MI R 98-99, 136
- MI R 98-99F, 145
26
0.
CARCINOMAS
DEL TRACTO URINARIO
Aspectos esenciales
G
El carcinoma vesical ms frecuente es el urotelial, siendo el tabaco el principal factor de riesgo.
(
El carcinoma escamoso se relaciona con la esquistosomiasis (5. haemafobium).
(
El adenocarcinoma vesical se relaciona con el antecedente de exLrofia vesical.
(
El carcinoma papilar superficial y el carcinoma in situ (OS) son muy recurrentes.
(
Clnica ms frecuente del carcinoma urotelial: hematuria, ms tpico con cogulos.
m
Cuando se trata de un carcinoma in situ: sntomas rritativos (polaquiuria, disuria, tenesmo, etc).
m
El mejor mtodo para la estadificacin local es la reseccin transuretral.
il
Prueba ms sensible para el diagnstico de OS: citologa urinaria.
(
Conducta ante un ClS: tratar con bacilo Calmette-Guerin (BCG ) y revisiones (cistoscopia y citologas).
G
Actitud ante un tumor superficial: reseccin transuretral . Posteriormente, revisiones (CiSloscopia y citologas).
G
Actuacin ante un tumor infiltrante (afectacin capa muscular): cistectoma.
6. 1 . Carci noma vesi cal
El carcinoma vesical es l a segunda neoplasia urolgica en frecuencia. Aparece ms frecuentemente en varones
(2-3 : 1 ) y ms en poblacin blanca que negra. Su edad de mxima incidencia se sita entre los 60 y 70 aos. De
el los, el 90% son carcinomas transicionales, el 8% escamosos y el resto adenocarcinomas. El epitelio uroteJ i al
recubre el tracto uri nario desde las papilas calicial es hasta l a uretra prosttica, ambas incl usive. En cualquiera
de estos niveles pueden desarrollarse los tumores uroteliales, correspondiendo la mayora a la vejiga (ms del
90%) y, ms raramente, al tracto urinario superior (5%) o la uretra (1 %).
Entre los factores etiolgicos (Tabla 1 2), se i mpl i can las ami nas aromticas, presentes en las industrias textiles,
qumicas y del caucho. El humo del tabaco es el principal factor de riesgo (50-60% aparecen en fumadores),
aumentando el riesgo a mayor consumo. Tambin pueden jugar un papel importante los edulcorantes artificiales
(sacarina, ciclamato), l a ciclofosfamida, los acetiladores lentos (mayor riesgo) y muchas otras posibles etiologas.
La infestacin por Schistosoma haematobium aumenta la incidencia de carcinoma escamoso vesical, as como
la presencia de i nfeccin crnica o catter vesical permanente.
Aminas aromtlcas {2-naftllaminal; tabaco, industria
textil, industria del caucho, colorantes
Fenacetinas crnicas Translclonales (9)
Sacarina, ciclamato Mejor pronstico
Ciclofosfamida (acrolelna)
Tabaco: ortofenoles, trlptfano
Schfsrosoma haematobium
Escamosos (8%)
litiasis, infecciones, catteres
Cistitis glandular
Adenocarclnomas (2%)
Extrofa vesical
Tabla 12. Factores etiolgicos de l os carcinomas del tracto urinario
El adenocarcinoma primario vesical es un tumor raro, aunque es el que
se ha visto asociado a l a extrofia vesical con mayor frecuencia.
RECUERDA
No hay que confundir Schisrosoma haemarobium con Schislsoma
mansoni, que produce hipertensin portal.
Histologa e historia natural
Haciendo referencia al carcinoma de clulas transicionales, hay que di
ferenciar tres formas de la enfermedad con comportamiento, pronstico
y tratamiento completamente distinto (Tabla 1 3 y Figura 21 ). El 70% de
los tumores vesicales se presentan como tumores papilares de crecimien
to principalmente endocavitario y frente de i nvasin nico. Un 1 0% son
slidos, con invasin tentacular en profundidad y extensin l i nftica y vas
cular temprana. El 20% restante son formas mixtas. El primer grupo suele
corresponder a tumores superficiales de bajo grado histolgico, mientras
que los slidos, con mayor frecuencia, son tumores infiltrantes de grado
hi stolgico ms elevado. la principal caracterstica de los tumores papi
lares superficiales es la recurrencia, que ocurre en un 50-75%, segn el
grado y estadio. El 25% recurrirn y progresarn en grado y estadio, y ni
camente e1 1 5% acabar desarrollando un tumor i nfiltrante o metastsico.
la mayora de los tumores infi ltrantes se encuentran confinados a l a
vejiga en el momento del diagnstico, y slo un 20-25% presentan
extensin ganglionar o metastsica. El 50% desarrollarn metstasis a
distancia, a pesar del tratamiento (MI R 04-05, 259).
El tercer grupo que merece mencin aparle es el carcinoma in situ. A
pesar de encontrarse l i mi tado al urotelio, por lo que es superficial, est
formado por clulas poco diferenciadas con displasia grave. Tiene una
alta tasa de recidiva y progresa hacia tumor i nfiltrante en el 50-75% de
los casos. Este mal pronstico le confiere un carcter completamente
distinto del carcinoma in siw de otras regiones, en las que se considera
el estadio i ni cial de la enfermedad tumoral. El carcinoma in situ puede
estar asociado a focos de carcinoma superficial (26%) o nfiltrante (60%)
o bien encontrarse de forma aislada, siendo generalmente multi focal
tanto en vejiga como en otras puntos del uratelio (MI R 07-08, 1 02).
Tis: carcinoma in situ (plano)
Ta: carcinoma papilar no infltrante
T1: tumor que invade tejido conjuntivo subepitelial
T2: tumor que invade msculo
- T2a: tumor que invade la mitad interna
- T2b: tumor que invade la mitad externa
T3: tumor que invade tejido perivesical:
T3a: microscpicamente
- T3b: macroscpicamente (masa extravesical)
T4: tumor que invade prstata, tero o vagina
T 4b: tumor que invade pared plvica o pared abdominal
Nx: metstasis ganglionar regional desconocida
NO: ausencia de metstasis ganglionar regional
N1: metstasis a un solo ganglio entre 2 y 5 cm
N2: metstasis en un ganglio mayor de 5 cm o mltiples no mayores de 5 cm
N3: metstasis mayores de 5 cm
MO: no metstasis
Ml: metstasis a distancia
Tabla 13. Estadifcacin del carcinoma vesical
UrOI09a _
1. Mucosa --Ta,Ts 4. Grasa -- T3
2.5ubmucosa J T1
3. Muscular --1
5. rganos vecinos _ T4
Figura 21. Esquema de la estadifcacin del tumor vesical
Se puede encontrar en el mbito vesical disti ntas lesiones benignas que
no se asoci an con el desarrollo de cncer: los nidos de von Brunn, l a
cistitis qustica y gl andul ar origi nados en procesos inflamatorios o rrila
tivos crni cos, y que probablemente sean disti ntas mani festaciones de
un mismo proceso, aunque pueden plantear el diagnstico diferencial;
(ocasionalmente se han descrito adenocarcinomas vesicales asociados
a l a cistitis glandular). Olras lesiones benignas seran el adenoma nefro
gnico, el plipo simple, el papiloma i nvertido o el papiloma velloso.
Di agnstico
la hematuria macroscpica o microscpica monosi ntomtica es el ha
l l azgo ms frecuente, presente en el 75% de los pacientes (MIR 06-07,
96) . la presencia de mi crohematuria asi ntomtica, descubierta durante
estudios de cribado, slo se relaciona con enfermedad significativa en
menos del 2% de los casos. Pueden encontrarse sntomas irritativos
(escozor, polaqui uri a, tenesmo) en el 25-30%, solos o acompaando a
la hematuria. la presencia de un sndrome cisttico no justificado por
i nfeccin o li ti asi s debe hacer sospechar la presenci a de un carcinoma
vesical, especi al mente por su asociacin con el carcinoma in situ (MIR
00-01 F, 1 44). Con menor frecuencia, el paciente consulta por dolor en
flanco por obstrucci n ureteral, plvico o por edema en miembros in
feriores (extensin l i nftica). la exploracin fsica suele ser i rrelevante,
salvo en la enfermedad avanzada.
las citologas uri narias son una prueba senci lla y fiable que debe ser
real i zada en todos los casos de hematuria asi ntomtica o sospecha de
tumor vesical. Su sensibilidad depende del grado de diferenciacin
del tumor vesical, alcanzando el 75-1 00% en tumores de al to grado
y carcinoma in situ, siendo en este l ti mo un mtodo diagnstico ms
rentable que la ecografa, la Te, la urografa o la biopsia mltiple. Son
especi al mente tiles en el segui miento de pacientes sometidos a resec
cin transuretral en combinacin con la ci stoscopi a.
Entre l as pruebas radiolgicas destacan l a ecografa (con una sensibi l i
dad del 80%, pera poco til para el diagnstico de las neoplasias del
tracto uri nari o superior (Figura 22]) y la urografa intravenosa (capaz de
detectar l a presenci a de tumor en el 60% de los casos).
27
Manual eTO de Medicina y Ciruga, B.a edicin
En l a urografa i ntravenosa, adems de la presencia de defectos de
replecin, se puede sugerir el diagnstico de tumor vesical, rigidez
y falta de distensi bi l idad vesical, la obstruccin de un urter o el
desplazamiento de l a vejiga, entre otros. En caso de dudas sobre el
tracto urinario superior, se recurrir a l a pielografa retrgrada en el
momento de realizar l a ci stoscopia.

sta es fundamental para la eva


luacin del tumor vesical; puede real i zarse bajo anestesia local cuan
do existan dudas con las pruebas reali zadas previamente, pero si el
diagnstico de presuncin es firme, y dado que en todo tumor debe
realizarse reseccin transuretral para evaluar el grado de infi ltracin,
se puede esperar a tener al paciente en qui rfano bajo anestesia gene
ral o raqudea para practicarla (MIR 00-01 F, 1 43). En un 1 0% de los
casos de ci stoscopia con ci tologas positivas no se encuentra tumor
en l a vejiga, lo cual puede ser debido a l a presencia de carcinoma in
situ, tumor en vas urinarias al tas, a un carcinoma ductal de prstata
o a un falso positivo de la prueba (generalmente por inflamacin de
la pared vesical o por tratami ento concomitante con radioterapia o
qui mioterapia endovesical).
La Te se utiliza para la estadificacin de l a enfermedad infi ltrante, ya
que aporta informacin l i mitada sobre la infi ltracin tumoral. La RM
consigue mejores imgenes de la cpula vesical por sus cortes sagita
les, pero no aporta mayor informacin que la TC
Tratamiento
Todo el proceso di agnstico va encaminado a establecer si el tumor
vesical es superficial o infiltrante, ya que el tratamiento vara radical
mente en funcin de este hecho.
Los tumores superficiales son manejados mediante reseccin transu
retral . Dada la alta frecuencia de recidivas, la mayora se tratan poste
riormente con i nstilaciones endovesicales (quimioterapia o i nmunote
rapia local) que disminuyan la aparicin de nuevos tumores. Entre los
qui mi oterpicos empleados se encuentran l a mitomicina, la tiotepa, la
adriamicina o la epirrubicina. Con todos ellos se consigue reducir las
recidivas al rededor de un 20%.
La i nmunoterapia endovesical con BCG (bacilo Cal mette-Guerin) es,
sin duda, l a ms eficaz, di smi nuyendo el porcentaje de recidivas en
28
un 40%, siendo tambin tratamiento de eleccin en el carcinoma
in situ, con una eficacia del 70% (MI R 05-06, 1 03; MI R 02-03, 1 79;
MI R 00-01 , 1 1 8). Si n embargo, no se uti li za en los tumores vesicales
como primera lnea, reservndose para los tumores de ri esgo o con
mlti ples recidivas, ya que tambin se acompaa de mayor nmero
de efectos secundarios y de complicaciones, entre los que se en
cuentran cistitis febril, sndrome pseudogripal, y las ms graves de
sepsis, prostatitis granulomatosa, neumonitis e i ncl uso muerte. Estos
cuadros precisan tratamiento tuberculosttico completo al menos de
tres a seis meses.
En l a enfermedad i nvasora O i nfi l trante, tras l a reseccin transure
tral para evaluar la afectacin parietal, el tratami ento de eleccin
es l a ci stectoma radical. La qui mioterapia sistmica, adyuvante o
neoadyuvante, no parece mejorar los resultados de la ciruga ai sl ada.
Igualmente se puede deci r de l a radioterapia, que ni camente ha de
mostrado i ncrementar el tiempo libre de recidiva local, si n aumentar
la supervivencia.
La qui mi oterapia combinada con M-VAC (metotrexato, vinblastina,
adriamicina, cisplatino) se reserva para pacientes en los que existe
afectacin ganglionar o metastsica, con respuestas completas en el
1 5-30% y parciales en el 30-40%. Tras la cistectoma, los urteres son
derivados generalmente a segmentos intestinales o a piel, pudiendo
realizarse estomas no continentes o reservaras continentes di recta
mente al remanente uretral.
RECUERDA
la adriamicina (doxorrubicina) es un quimioterpico que puede produ
cir cardiotoxicidad.
6.2. Tumores del tracto uri nari o
superi or
Entre el 2-5% de los tumores urotel i al es se encuentran l ocal i zados
entre los cl i ces y los ori fi ci os ureterales. En su etiologa estn
i mpl i cados los mi smos factores que para el carci noma vesi cal , a
los que habra que aadi r la nefropata por abuso de analgsicos
(fenacetinas) y la nefropata de los Bal canes. En el 70-80% de los
pacientes aparece hematuri a macroscpica, siendo el dolor c
lico por obstruccin ureteral l a segunda queja en frecuenci a de
apari ci n.
RECUERDA
la hematuria tambin es l a manifestacin ms frecuente del hiperne
froma.
Diagnstico
La si stemti ca di agnstica es bsicamente l a mi sma que para el
tumor vesi cal . Es decir, ecografa (detectar hi dronefrosis del l ado
del tumor), UIV (se ver un defecto de replecin no compatible
con un cl cul o o una anul aci n funci onal de ese si stema excretor),
y Te para la estadificacin (Fi gura 23) ( MI R 98-99, 1 36; MI R 98-
99F, 1 45) .
Figura 23. Ecorafia. Tumor vesical con zonas calcifcadas
En estos tumores, la citologa uri nari a aumenta su eficacia si se ob
tiene de forma selectiva, cateterizando el urter del lado afectado.
Otros elementos de di agnstico son las biopsias por cepillado y l a
Varn de 57 aos de edad, fumador, que consulta por hematuria terminal, polaquiu
ria, urgencia y dolor miccional. Presenta citologas urinarias positivas de carcinoma
urotelial, y el estudio anatomopatolgico tras la reseccin transurelral e de carci
noma in silu difuso, con intensa inflamacin crnica. El tatamiento estndar s:
1 ) Instalacin del bacilo d Calmette-Guerin.
2) Cistectoma radical.
3) Instilaciones con mitomicina.
4) Quimioterapia con cisplalino.
5) Antiinllamatorios no esteroideos ms quinolonas durante seis m.
MIR 05-06, 1 03; RC: 1
UrOIOga
ureteropieloscopia, elemento que, adems de diagnstico, puede
tener un fin teraputico en tumores de pequeo tamao y aspecto
superfi ci al (Figura 241.
UH80OI+ MEH8\UI8
Sospecha d. tumor uroteUal
+ _LLLl
"

Ll\OOgI8S+ lIuPD8SOP08gPD
UIV
t
NOCOHCUyED\E
8I8\UHOIVESC8l
t
t
LOHCuyEH\E
8I8\UHOIVESlC8
t
LS\OSCO8 E __:..
K

"
'': __
6 t t t t
bOS8VPSlC8 E S 212
F,1
6 t t
bL LS\EC\OH8 LUHlOLLIL5] bOSDECD8I\U0OI
I8C\O UID8IIO SUElOI
UIE\ElOllEDOSCO8
EOgI8T8 IE\Ig8O8
KEVSODES
LS\OSCOl8
+ L\OlOg8
KECllV81SL1OH8 KOV8 -
Ll\OOg8SSEEC\V8S
LE8OOUIE\EI8!
Ftura 24. Algoritmo diagostico-teraputko en tumor de vejiga (MIR 03-,82)
Tratami ento
El tratamiento estndar es l a nefroureterectoma radical con extirpacin
de un rodete perimetico vesical; tal extensin es necesaria por l a ten
dencia a la recidiva de estos tumores. Es planteable, si n embargo, el
empleo de tratamientos conservadores en caso de tumores papilares,
no infiltrantes, nicos y de pequeo tamao.
Es necesario un seguimiento posterior de la vej iga y el rin contra la
teral, ya que un 20-30% desarrollarn un tumor vesical metacrnico y
un 2% en el sistema colector contralateral .
Casos clnicos representativos
Mujer de 63 aos que es diagnosticada d carcinoma de clulas escamosas del I
gono vesicl, con invasin de l capa muscular. Cul s su actitud leraputica
en este c?
1 ) Radioterapia externa con 7.00rads.
2) Quimioterapia adyuvante, seguida d cistectoma radical.
3) Reseccin transureral, seguida de inmunolerapia intravesical (BCG).
4) Cistectoma radical con etirpacin de cara anterior de vagina.
5) Radioterapia externa, seguida de quimioterapia con cisplatino.
MIR 03-04, 82; RC: 4
29
Manual CTO de Medicina y Ciruga
,
8 ... edicin
30
Un paciente de 6 aos acude a consulta por sndrome miccional de dos meses de
evolucin. Refiere que sU MAP le ha dado tratamiento antibitico y le ha realizado
cultivos que han sido negativos. Se le realiza una ecografa y una flujometra que son
normales y una urografa intravenosa que no evidencia alteraciones. En la cistosco
pia no se observan lesiones intravesicales. las citologas urinarias son sugestivas de
malignidad. Seale la incorrecta
1 ) El tratamiento de eleccin ser la inmunomodulacin vesical.
2) Es imprescindible la realizacin de biopsias para confirmacin del diagnstico.
3) Si recidiva tras las instilaciones, sedeben repetir los ciclos dos veces ms.
4) la cistectoma es el tratamiento final en muchos de estos pacientes.
5) A priori no se encontrar infiltracin de la capa muscular.
RC: 3
Casos clnicos representativos
Una paciente de 58 aos acude por polaquiuria de dos aos de evolucin, nicturia
y dolor hipogstrico. Niega haber visto sangre en la orina y es fumadora. Seale la
respuesta correcta:
1 ) Se deben realizar biopsias a esta paciente.
2) Una dstoscopia permitir confirmar el diagnstico.
3) los tratamientos empleados hasta el momento han demostrado una gran eficacia.
4) Muy probablemente los cultivos sern positivos.
5) En todos los casos se detectarn > 1 0 hemates/campo en el sedimento de orina.
Re: 1
0/.
TUMORES TESTICULARES
Ontnti(In
MIR
Estc tema es sencillo
y rentable siempre
que se seleccione
lo realmente importante:
la clnica y el diagnstico.
Es fundamental la revisin
profunda de los Desgloses.
En cuanto al tratamiento, dado
que no existe un protocolo
universalmente aceptado,
es mejor conocer ideas
generales.
1 Preguntas
- MIR 09-10. 102
- MIR 06-07, 94
- MIR 05-06. 107
- MIR 04-05, 107
- MIR 03-04, 145
- MIR 02-03, 186
- MIR 01 -02, 108
- MIR 00-01 F, 142
- MIR 99-00, 186
- MIR 99-00F, 145
- MIR 98-99F, 218
m
o
[
Aspectos esenciales
El tumor testicular es la neoplasia slida ms frecuente en el varn joven.
la tasa de curacin es mayor del 90%.
El tumor testicular ms frecuente es el seminoma. No obstante, sto es muy variable segn la edad del pa
ciente.
Una masa testicular por encima de los 50 aos debe hacer penslr en un linfoma.
El tumor tpico de las disgenesias gonadales es el gonadoblasloma.
la clnica ms frecuente es una masa escrotal indolora.
El seminoma nunca produce afetoprotena.
Como tratamiento, la orquiectoma por va inguinal se realiza en todos los casos.
El seminoma es radiosensible. Por ello puede utilizarse radioterapia como tratamiento en lo primeros esta
dios. Si se trala de un estadio avanzado, se empleara quimioterapia.
los lumores no seminomalosos s tratan con orquiecloma + quimioterapia. No obstante, si es un tumor
l imitado al testculo, puede plantearse la vigilancia tras la orquiectoma.
7. 1 . Eti ol oga y epi demi ol oga
Los tumores testiculares suponen el ' 2% de las neoplasias en varones, y son las neoplasias slidas ms frecuen
tes entre los 20 y 35 aos. Presentan mayor incidencia a mayor desarrollo de la sociedad, y tambin en la raza
blanca ms que en las dems. La tasa de curacin es superior al 90%.
El 95% de ellos proceden de clulas germinales y, aunque globalmente el semi noma es el ms frecuente, la in
cidencia vara segn el grupo de edad considerado. El 5% restante se reparte entre tumores del estroma gonadal
(1 2%), ! i nfamas (1 %), gonadoblastomas (clulas germinales y del estroma) metstasis y otros.
Los factores de riesgo para el desarrollo de tumor testi cul ar son: teste hi poatrfico, sndrome de Kl i nefelter,
hi storia fami l i ar de neoplasias testi cul ares, tumor previo en el otro teste y criptorqui di a. Los testculos no
descendidos tienen mayor riesgo de desarrollar tumores, y esta probabi l i dad aumenta si l a situacin del
teste es intraabdomi nal . Asimismo, el testculo contra lateral, aunque de local i zaci n escrotal, tiene una
i ncidenci a mayor; de hecho, el 20% de los tumores en paci entes con criptorquidia se desarrollan en el
testculo no criptorqudico. Por estas razones, l os testculos no descendidos deben descenderse, preferi
blemente entre el pri mer y segundo ao de edad para faci l i tar su seguimiento y, con el lo, la deteccin de
un eventual tumor.
Una vez alcanzada la pubertad, un teste criptorqudico no descendido probablemente deba exti rparse, dado que
estos testculos pierden l a capacidad de espermatognesis y conservan su potencial malignizante. No obstante,
hay autores que prefieren no extirparlos, siempre y cuando se puedan descender a la bolsa escrotal, ya que an
pueden mantener su funcin endocrina (secretora de testosterona) (MIR 9900, 1 86).
Otros factores que se pueden encontrar relacionados son las hernias inguinales infantiles (no demostrado) y
la orquitis urliana secundaria al paramixovirus causante de la parotiditis (siempre que haya producido atrofia)
y, entre los factores txicos, la exposicin a radiaciones, fuentes de calor, productos para teido del cuero y
31
Manual eTO de Medicina y Ci ruga, 8.3 edicin
estrgenos ntra tero durante el primer trimestre del embarazo (MI R
98-99F, 21 8) . Aunque entre un 8 y 25% de los pacientes presentan
historia de traumatismo testicular, todos los autores parecen estar de
acuerdo en que ste supone ms bien el motivo por el que se descubre
una masa escrotal, y no su origen.
7. 2. Anatoma patol gi ca
Los tumores testiculares se suelen originar como una masa testicular
intraparenqui matosa. A partir de ah, pueden quedarse localizados o
metastatizar. La localizacin ms frecuente de metstasis son los gan
glios retroperitoneales (casi siempre es el primer escaln en la disemi
nacin) posteriormente puede aparecer la afeccin mediastnica, de
vsceras abdominales y de pulmn.
Hay que tener en cuenta que el testcul o izquierdo drena a los ganglios
paraarticos y prearticos a nivel de L2. El derecho drena a ganglios
interaortocavos, precavas y prearticos, y tambin a nivel del hi l i o re
nal . La diseminacin hematgena es menos frecuente, salvo en el co
riocarcinoma, va vasos espermticos, siendo los puntos ms habituales
de metstasis: pul mn, hgado, hueso y SNC, por orden de frecuencia.
Los tumores testiculares se dividen en dos grupos: tumores que no deri
van de las clulas germinales (5%) y tumores que derivan de las clulas
germinales (95%); a su vez, estos ltimos se dividen en seminomatosos
y no seminomatosos (Tabla 1 4) .
TUMORES DE CHULAS GERMINALES TUMoRES DEL ESTRoMA
T de un tipo
$eminoma
- Tpico
- Anaplsico
- Espermatoctico
Carcinoma embrionario
PoHembrioma
Tumor del co vitelina (seno endodrmico)
Coriocarcinoma
Tumores clulas Leydig
Teratoma:
Tumores clulas SertoH
- Maduro
Tumores estructuras
- Inmaduro
gonadales primitivas
- Con transformacin maligna
Tumores de m6s de un tipo hg
Teratocarcinoma
Otros
Tumores germinales + e'
32
Gonadoblastoma
Tabla 14. Tumores de testkulo. Clasifcacin histolgica
Serinora. Puede aumentar el tamao testicular hasta 1 0 veces si n
distorsionar su morfologa. Ocasionalmente se presenta extrates
t cular en mediastino (l a ms frecuente), retroperitoneo, o regin
pineal de forma primaria. Se han descrito tres tipos histolgicos:
seminoma tpico, anaplsico y espermatoctico (ms frecuente en
individuos mayores de 50 aos). El seminoma puro no es productor
de marcadores tumorales, pero hasta en un 1 5% de los casos pue
den aparecer clulas del sincitiotrofoblasto, produciendo elevacio
nes de l a p-HCG.
Carcinoma embrionario. Tiende a metastatizar de forma temprana.
Forma masas ms irregulares y heterogneas que otros tumores.
Tumor del seno endodrmico. Este tumor fue descrito i ni ci al mente
como una forma rara de tumor infantil. Se encuentran elementos de
saco vitelina en el 38% de los tumores testiculares del adulto.
Coriocarcinora. En el estudio histolgico tiene que contener ele
mentos de sincitiotrofoblasto y citotrofoblasto para ser considerado
como tal. En el momento del diagnstico, generalmente existen me
tstasis a distancia (suelen ser va hematgena) y un tumor primario
testicular pequeo. Raramente es puro (MIR 06-07, 94).
Teratoma. Por definicin, se encuentra formado al menos por dos capas
distintas de clulas germinales (endodermo, mesodermo o ectodermo).
Teratocarcinora. Tumor mi xto con reas de teratoma y de carcino
ma embrionario. Un 64% tiene tambin reas de semi noma.
Conadoblastoma. Contiene grandes clulas simi lares al seminoma
y otras menores, como clulas de Sertoli inmaduras o de l a granulo
sa. Se asocia preferentemente a las gnadas disgenticas y estados
intersexuales.
Tumores de clulas de Sertoli. Forman reas tubulares si mi l ares a
los tbulos del testculo prepuberal normal. De comportamiento be
nigno, es excepcional que metastaticen.
RECUERDA
El lumor de clulas d Serlol i se ha asociado al sndrome de Peutz
Jeghers (vase Apartado de Sndromes de poliposis, en la Seccin de
Digestivo y ciruga general.
Tumores de clulas de Leydig. Pueden verse cristaloides en su cito
plasma. Generalmente son benignos. Pueden ser hormonal mente acti
vos, produciendo pubertad precoz o feminizacin (MIR 99-00F, 145).
Linfora. Tanto como localizacin secundaria o como origen pri
mario (menos frecuente), una masa testicular en un varn mayor de
50 aos sugiere en primer lugar el diagnstico de l i nfoma. Tras la
orquiectoma o l a biopsia testicular para el di agnstico de certeza,
el tratamiento no vara respecto a los l i nfomas habituales.
7. 3. Cl ni ca
La manifestacin ms frecuente es como masa escrotal indolora. Con
mucha menor frecuencia, el motivo de consulta est originado por la
presencia de metstasis ganglionares (masas supraclaviculares o abdo
minales (MIR 09-10, 1 02), o efectos endocrinos (ginecomastia, puber
tad precoz).
EI 1 0 % de los tumores se presentan como escroto agudo en la urgencia.
RECUERDA
Exislen algunas enfermedades, como la sarcoidosis, que aumentan el
tamao testicular sin existir un tumor.
7.4. Di agnstico
La exploracin fsica de una masa indolora, si n signos inflamatorios y
de l argo periodo de evolucin, debe hacer sospechar un tumor testicu
lar (Tabla 1 5) .
Un elemento i mportante, tanto para su diagnstico como para el segui
miento, son los marcadores tumorales: a-fetoprotena (AFP) y fraccin
p de l a gonadotropina corinica humana (P-HCG) (Tabla 1 6).
pTls
pTl
pT2
pTl
pT4
N1
N2
N3
M1.
M1b
Sx
SO
51
52
53
I ntratubular
Testculo y epididimo sin invasin vascular/linftica
Testculo y epiddimo con invasin vascular/linftica o tnica
vaginal
Afectacin de cordn espermtico
Escroto
Ganglios menores de 2 cm
Ganglios entre 2-5 cm
Ganglios mayores de 5 cm
Metstasis en ganglios no regionales o pulmn
Metstasis viscerales no pulmonares
Marcadores tumorales sricos no disponibles
Niveles de marcadores normales
lDH < 1.5 x n; y p-HCG < 5.000; Y AFP < 1 .000
lDH entre 1,5 x n y 10 x n o p-HCG entre 5.000 y 50.000
o AFP entre 1 .000 Y 10.000
LDH> 10 x n o j-HCG >50.000 o AFP > 1 0.000
Tabla 15. Estadifcaci6n clnica y patolgica
Tumor con una palabra aumenta j-HCG
Coriocarcinoma
- Seminoma (en la mujer, el equivalente es el disgerminoma)
Tumor con dos palabras aumenta a-fetoproteina
- Carcinoma embrionario
- Tumor del seno endodrmico
Tabla 16. Marcadores tumorales en tumores germinales
(vlidos para ovario y testiculo)
La AFP es si ntetizada por clul as del saco vitel i no y, por tanto, est
presente en tumores de saco vi telino o seno endodermal, y en los
carcinomas embrionarios. El semi noma nunca produce AFP ( MI R
04-05, 1 07; MI R 02-03, 1 86). En cual qui er caso, se debe tener en
cuenta que l a AFP es un marcador i nespecfico, y se podra en
contrar elevado en enfermedades hepticas benignas o mal i gnas,
algunas neoplasias pancreticas y de l a va bi l i ar o en la ataxia
telangiectasia.
RlECU
F
pERDAb
I l
' -
d
.
.
I
.
d
i
'
a A t len se e eva en IqUl o amOlotlCO en a teraClones e CIerre
del tubo neural. Ver Ginecologa.
La fraccin p de la HCG es producida por las clulas del si ncitiotrofo
bl asto presentes en el coriocarcinoma, y tambin de forma ocasional y
de forma ai slada, en algunos semi nomas (MI R 00-01 F, 1 42).
En conjunto, el 70% de los tumores testiculares producen algn marca
dor, luego existe hasta un 30% de tumores con marcadores negativos
al diagnstico.
La vida media de l a a-fetoprotena es de siete das, frente a tres das
de la -HCG. Este dato es i mportante a la hora de valorar la posible
UrOIOga
presencia de enfermedad residual, ya que la elevacin persistente de
uno de estos marcadores despus del tratamiento supone l a existencia
de tumor no el i mi nado.
La ecografa testicular es un mtodo sencillo y fiable para l a diferen
ciacin entre masas slidas y qusticas, y su localizacin exacta i ntra
testicular o dependiente de los anejos. Cuando, a pesar de los marca
dores, los datos ecogrficos son sugerentes de tumor, est indicada la
exploracin qui rrgica a travs de una incisin i ngui nal , para evitar la
posibi lidad terica de implantes tumorales en l a piel escrotal y tener
mejor control de pedcul o vascul ol i nftico a nivel del cordn esperm
tico (MI R 01 -02, 1 08).
Si la exploracin confirma l a presencia de una masa, el testculo debe
ser extirpado (orquiectomfa radical).
La evaluacin de la extensin tumoral se completar mediante TC to
racoabdominal. As se constatar si la enfermedad est l i mi tada al tes
tculo (estadio 1), o existe afectacin de ganglios i nfradiafragmticos
(estadio 1 1 ), o bien si hay incluso extensin supradiafragmtica o a r
ganos slidos (estadio 1 1 1). El sistema de estadificacin utiliza mltiples
variaciones, pero quiz la clasificacin ms aceptada sea l a expuesta
anteriormente en la Tabla 1 5.
7. 5. Di agnstico diferenci al
Aunque frecuentemente l a existencia de un tumor testi cul ar no plan
tea dudas di agnsticas, existen una serie de patologas testi cul ares
que, junto con los tumores, pueden presentarse en al gn momen
to dentro del cuadro genrico que se denomi na "escroto agudo",
caracterizado por el aumento doloroso de volumen del contenido
escrotal, acompaado o no de signos i nfl amatorios (Tabla 1 7 y Fi
gura 25).
Orquiepididimitis: suel en presentarse con dol or i ntenso, enro
jeci miento cutneo, fiebre, y a veces si ntomatologa mi cci onal .
En l a exploracin, la elevacin del testcul o (si gno de Prehn)
di smi nuye el dolor (signo de Prehn positivo). En su etiologa se
deben considerar grmenes de transmi si n sexual en pacientes
adul tos menores de 35 aos, y uropalgenos (E. coli) si superan
esla edad.
Torsin del cordn espermtico: suele aparecer en la infancia o
la adolescencia, con dolor de aparicin brusca y signos cutneos
inflamatorios crecientes a medida que progresa el cuadro. El teste
se encuentra horizontal izado, y ocasional mente puede palparse la
espiral del cordn torsionado. En este caso, la elevacin del testcu
lo i ncrementa la sensacin dolorosa.
Hidrocele y espermatocele: son dos cuadros que raramente se
presentan de forma brusca y con dolor agudo, fcil mente diferen
ciables por la exploracin y su transi l umi nacin positiva y, ante l a
duda, mediante ecografa.
Orquitis
Torsin del cordn espermtico
Tumor testicular
Polaridad conservada
Prehn MEJORA el dolor
Testculo horizontalizado
Prehn EMPEORA el dolor
Masa palpable indolora
Tabla 17. Diagnstico diferencial de los tumores testiculares
33
Manual eTO de Medicina y Ciruga, 8.a edicin
Figura 25. Piez de tumor testicular
7.6. Tratami ento
Hasta la fecha no existe un protocolo nico de tratamiento, pudiendo
variar incluso de un centro a otro. Siempre se realizar orquiectoma
radical va i ngui nal . Posteriormente, el patlogo informar de la estir
pe histopatolgica. A continuacin, el estudio de extensin mediante
Te toracoabdominoplvica y nuevos marcadores postorquiectoma. En
funcin de la histologa y del estadio, se asignar un tratamiento a cada
enfermo.
RECUERDA
La orquiectoma en el cncer de testculo e va inguinal. En el cncer
de prstata, va ertl.
RECUERDA
El cisplatino prouce vmitos con mucha frecuencia. Otro efecto secun
dario e su nefrotoxicidad.
Estadio lIa-lIb. En este caso, el tumor ya est extendido a gangl ios
retroperitoneales, y requiere, por tanto, tratamiento agresivo. Se
dispone de dos opciones: radioterapia sobre las cadenas afectas
(teniendo en cuenta el teste afectado, se irradiar a unas cadenas
ganglionares u otras), o qui mioterapia BEP (cisplatino, etopsido y
bleomicina). Ambos obtienen resultados muy si mi l ares
Estadio 11e-1II (estadios avanzados). El tumor tiene metstasis gan
glionares retroperitoneales superiores a 5 cm, O afectacin supra
diafragmtica o de vsceras slidas. La radioterapia deja de ser una
opcin teraputica. La quimi oterapia (BEP) es l a nica posibilidad,
presentando una tasa de curacin de al rededor del 80%.
Tumores no semi nomatosos
Estadio 1. Existen tres posibilidades teraputicas tras la orquiectoma:
Observacin y seguimiento peridico. Tasas de recadas de un 20%.
li nfadenectoma retroperitoneal de estadificacin, pues as se
tiene certeza del estadio y se reduce a un 1 0% las recidivas (en
Europa no est extendida esta prctica).
Qui mi oterapia profilctica (cisplatino), y as se reducen al 5%
las recidivas.
En aquellos casos en que exista invasin vascular en la pieza de
orquiectoma, l a tasa de recidivas asciende al 50%. Parece lgico,
en estos casos, inclinarse de entrada por una de las dos ltimas op
ciones. La tasa de curacin alcanza el 98%.
Estadio lIa-lIb. Hi stricamente se realizaba l i nfadenectoma retro
peritoneal completa como ni co tratamiento, pero ante tasas de
recidiva no desdeables, actual mente se indica qui mi oterapia de
i ni ci o (BEP). La tasa de supervivencia supera el 95%.
Estadio 11e-1II. Antes de la aparicin de l a actual qui mioterapia, la
supervivencia era del 51 0%. Actualmente, el esquema de trata
miento mayoritariamente aceptado es qui mi oterapia primaria.
El tratamiento de l a neoplasia testicular se resume en l a Figura 26.
TUMOR TESTICULAR
t
Orquiectoma radical
Seminoma
SEMINOMA NOSEMINOMA
Se caracteriza por su gran radiosensibilidad, de ah que la radioterapia
haya sido la base del tratamiento de estos tumores. Actualmente, l a
quimioterapia obtiene resultados si mi l ares.
34
Estadio l. El tumor tericamente est l imitado al testculo. No obs
tante, se sabe que hasta un 20% de pacientes presentan micro
metstasis a ganglios retroperitoneales, que en el momento del
diagnstico no se detectan. La presencia de micrometstasis se ha
relacionado con dos factores de riesgo: tumor de ms de 4 cm y la
i nvasin tumoral de la rete lestis. En estos enfermos se aconseja ad
mi nistrar radioterapia o qui mioterapia (cisplatino). Con sto, frente
a los pacientes que optaron ni camente por observacin, la recidiva
desciende del 20 al 5%.
t t
Observacin
Si FR :
> 4 cm reretestis
Radioterapia
Quimioterapia
lIa,lIb
t
Radioterapia
Quimioterapia
t
la
Observacin
Linfadenectomra
(EEUUI
Quimioterapia
(UEI
SEMINOMA I NO SlO
Quimioterapia
t
tla,lIb
+++
t
invasin
Quimioterapia
vascular
o
linftica
Figura 26. Algoritmo de tratamiento de la neoplasia testicular
Masas residuales
Se define como masa residual l a existencia de conglomerados adeno
pticos tras tratamiento qui mioterpico o radioterpico. Cuando el tu
mor primario es un semi noma y existen masas residuales, la actuacin
a segui r ser: si la masa residual es inferior a 3 cm, tiene muy pocas
probabi lidades de contener tumor residual y no requiere ms que ob
servacin. Pero cuando es superior a 3 cm, se debe real i zar una PET
(tomografa por emisin de positrones), si existiera esta posi bilidad a
nivel tcnico, pues detecta con una alta sensibilidad y especificidad l a
presencia de tumor residual . Si no se dispone de una PET o si sta es
positiva, se realizar ciruga de l a masa.
Cuando el tumor primario es un tumor no seminomatoso, se debe rea
lizar exresis de dicha masa siempre, con independencia del tamao.
La histologa de estas masas residuales, una vez extirpadas, puede ser:
tejido necrtico hasta en un 50% de las ocasiones, tumor viable en un
1 5% y teratoma en un 35% (stos, dejados a libre evolucin, pueden
convertirse en teratomas mal ignos o producir procesos compresivos
con su crecimiento) (Figura 27).
Un hombre de 31 aos de edad consulta por la presencia de una masa palpable en
el teste derecho, de un m de evolucin, no dolorosa. Su urlogo le realiza una
ecografa testicular, en la que se evidencia una lesin hipoe<oica, bien delimitada,
intratesticular. los marcadores tumorales a-fetoproteina y HCG son negativos. la
actitud m correcta de, entre las siguientes, sera:
1) Dado que los marcadores tumorales son negativos, se descarta neoplasia testicular
y requiere observacin.
2) Repetir la ecografa testicular en un plazo de tres meses.
3) Realizacin de una tomografa axial compularizada loraco-abino-plvica.
4) Biopsia transeseralal dcl tcstculo.
5) Orquiectoma radical y esperar resultado del patlogo.
MIR 05-06. 107; Re: 5
.
<3 cm

Observacin
MASA RESIDUAL
SEMINOMA
> 3 cm
+
+/- PET
-
- e ; "$

NOSEMINOMA
Exresis de la masa residual
50% necrosis 35% tratoma 15% tmor viable
Figura 27. Algoritmo de tratamiento de las masas residuales
Casos clnicos representativos
Qu diagnstico, entre los siguientes, es el ms probable en un nio de 8 aos con
signos inequvoco de pubertad precoz y que, en la exploracin, presenta una masa
en c testculo derecho de 2 cmde diimetrol
1 ) Tumor de clulas de leydig.
2) Sminoma.
3) Tumor del saco vitelina.
4) Teraloma.
5) Coriocarcinoma.
MIR 03-04, 1 45; Re 4
35
Orlentaclon
MIR
lo ms imJrtante de
estetema son los tipos de
rechazo, que s solap. .n con
Inmunologla. Se debe insislir
en el rechazo agudo, que
conviene repasar con las
preguntas de aos anleriOfe5.
7 Preguntas
- MIR 02-03, 178
- MIR 99-00F, 141
36
08.
TRASPLANTE RENAL
O
las causas ms frecuentes de insuficiencia renal crnica son la diabetes mellitus y las glomerulonefritis.
o
la clnica caracterstica del rechazo agudo es: fiebre, hipertensin y dolor en el rea del injerto.
G
El rechazo agudo produce oliguria, no poliuria.
8. 1 . I ndi caci ones
las dos enfermedades que ms comnmente abocan a una insuficiencia renal termi nal irreversible, tratable
mediante un trasplante renal, son la glomerulonefritis y l a di abetes mellitus i nsul i nodependiente.
Otras causas importantes son:
Poliquistosis renal.
Nefroesclerosis hipertensiva.
Enfermedad de Alport.
Nefropata IgA.
lupus eritematoso sistmico.
Nefroesclerosis.
Nefritis intersticial.
Pielonefritis.
Uropata obstructiva.
los mejores receptores son individuos jvenes cuyo fallo renal no se deba a una enfermedad sistmica que pue
da daar el rin trasplantado o causar la muerte por causas extrarrenales. General mente se suele mantener al
receptor en tratamiento con dilisis durante un cierto tiempo previo al trasplante.
8.2. Contrai ndi caci ones
las contraindicaciones absolutas son las siguientes:
Infeccin activa.
Enfermedad maligna que no pueda ser erradicada.
Sospecha de no cumplimiento teraputico del protocolo i nmunosupresor.
Glomerulonefritis activa.
Expectativa de vida reducida por enfermedad de base del paciente.
Presencia de anticuerpos preformados frente a antgenos del donante.
En referencia a las contraindicaciones relativas del trasplante renal se debe decir que stas se han ido modifi
cando a lo largo de los aos, al mejorar l a tcnica y los cui dados prequirrgicos y postqui rrgicos. En muchas
ocasiones, el trasplante plantea menos riesgo que una hemodilisis crnica.
Actualmente se consideran contraindicaciones relativas la edad avan
zada, l a oxalosis, la ami loidosis, la enfermedad i l i ofemoral oclusiva,
las anomalas del tracto urinario i nferior O las al teraciones psiquitricas
graves (MIR 99
-
00F, 1 4 1 ).
8. 3. Compl i caci ones
Las complicaciones que se pueden presentar son las siguientes:
Rechazo (Tabla 1 8) (MI R 02-03, 1 78).
Recurrencia de la enfermedad en el rin trasplantado.
RECHAZO INICIO PATOGENIA
urOloga
Complicaciones tcnicas. Complicaciones vasculares, hemorragia,
hi pertensin por estenosis de la arteria renal, trombosis venosa,
complicaciones del tracto uri nario, necrosis tubular aguda, l i nfo
celes.
Complicaciones no tcnicas. I nfecciones bacterianas y oportunis
tas en relacin con l a i nmunosupresin, hiperglucemias, compli
caciones gastrointesti nal es, hi perparatiroidismo y tumores (cncer
de piel y de l abi os, carcinoma in silU de crvix, l i nfomas no
Hodgkin; guardan relacin con el tratamiento i nmunosupresor).
Puede aparecer hi pertensin debida a enfermedad en los riones
ori gi nal es, como consecuencia de rechazo, por estenosis de l a
anastomosis de l a arteria renal o por toxicidad renal por ciclos
porina.
PA TRATAMIENTO
Minutos. das Ac. prefrmados Trombsis micrvascular Nefrectoma del injerto
Hd
CID Isquemia o infarto
Act. del complemento PMN en capilares
Dao endoletelial
Das Celular (+Ac) Vasculitis necrotiznte Bolos de esteroides
-
Respuesta 2. a Ag-HL Ac monoclonales
A
Semanas Celular (+Ac) Forma vascular: mediada por A Bolos de esteroides (la vascular suele ser resistente)
Infltrado de linfocitos Forma celular: tubulointersticial Ac monoclonales
Meses. aos Humoral y celular Intima arterial aumentada No hay; control de HTA
C Atrofa tubular
Glomerulopata
Tabla 18. Rechazo en el trasplante renal
1) Crisis hipertensiva.
2) Infecin respiratoria.
Un paciente de 35 aos, con insuficiencia renal crnica, secundaria a pielonefritis
crnica recibe un trasplante renal de cadver con el que comparla dos idenlidades
en A y 8 Y una en DR. Recibe tratamiento inmunosupresor con cidosporina A y cor
ticoides en dosis estndar. En el posoperatorio inmediato s oa buena diuresis, y
no es necesario el tratamiento sustitutivo con hemodilisis. En el 5." da de evolucin,
el paciente pesenta fiebre de 38", lA de 180/110, oliguria y disminucin en la con
centracin urinaria de sodio. El diagnstico ms probable sera:
3) Pielonefritis aguda del injerto renal.
4) Recidiva de su enfermedad renal.
5) Rechazo agudo del injerto renal.
MJR 02-03, 1 78; RC: 5
37
Oflentaclon
MIR
Tema poco preguntado hasta
la fecha. Se de tener una
idea general y aprender los
Aspectos esenciales.
, Preguntd<
MIR 07.8, lOS
. MIR 98-99f, 138
38
0.
U ROPATA OBSTRUCTIVA
G
la uropata obstructiva puede producir insuficiencia renal si no se resuelve a tiempo
.
(
El dolor suele estar presente en la obstruccin aguda. Sin embargo, en la crnica, es frecuente su ausencia.
(
Despus de resolver una uropata obstructiva puede producirse una fase de poliuria.
9. 1 . Caractersti cas
Detencin del flujo de orina en cualquier punto entre los clices renales y el exterior (Tabla 1 9) .
Su i mportancia reside en el desarrollo potencial de insuficiencia renal, por lo que tiene importancia l a obs
truccin urinaria bilateral o la uni late-
ral sobre rin nico funcionante.
Una obstruccin de ms de un mes
de duracin puede dar lugar a un
dao renal funcional y estructural
permanente.
RECUERDA
la uropata obstructiva puede producir glomerulonefrltis focal y
segmentarla.
INTRAlUMINAl INTRAPARIETAl
EXTRAPARIETAL EXTRAPARIETAl
(compreslon extnnseca) (dlsfunclon neuromuscular)
litiasis
Tumores (hipernefroma,
uroteliomal
Necrosis papilar
Cogulos
Estenosis congnita
Estenosis postinfecciosa
Estenosis postraumtica
Estenosis isqumica
Urter retrocavo
RiMn en herradura
Fibrosis retroperitoneal
. Tumores (prstata,
vejiga, ginecolgicos)
Hiperplasia prosttica
ligadura iatr6gena
de urteres
Tabla 19. Clasifcacin de la uropatfa obstructiva
9. 2. Cl ni ca (MIR 98-99F, 1 38)
La forma de presentacin depende de los siguientes factores:
Vejiga neur6gena
Vejiga automtica: lesin sobre
L1-L3
Vejiga tona: lesin bajo ll-L3
Disfuncin de la unin
pieloureteral
Reflujo vesicoureteral
Etiologa de la obstruccin. Presenta la clnica propia de la enfermedad de base.
Tiempo de evolucin. La aguda suele cursar con dolor (clico nefrtico), siendo l a crnica ms frecuentemen
te asintomtica.
Lugar de obstruccin:
Tracto urinario inferior (uretra y vejiga). Cursa con retraso para i niciar la miccin, disminucin de fuerza
y del tamao del chorro, goteo termi nal, hematuria, escozor al orinar, orina turbia, retencin aguda de
orina o i ncontinencia paradj ica ("miccin por rebosamiento").
Tracto urinario superior (urter y rin). Estos pacientes presentan dolor en el flanco (rin y urter proxi
mal), dolor en flanco con irradiacin a genitales (urter medio) o sndrome miccional (urter terminal) .
Despus de resolverse una obstruccin, sobre todo si es crnica, puede
producirse una fase de poliuria. Esto se debe a que, a nivel tubular,
cuando se ralentiza crnicamente el flujo urinario, se genera una in
sensibilidad a la ADH transitoria (diabetes inspida nefrognica), de ah
l a poliuria.
9. 3. Di agnsti co
El di agnstico de seguridad, la valoracin de la evolucin, y el pro
nstico son ecogrficos. Adems, son tiles la anamnesis y la explo
racin fsica, la radiologa simple (conveniente en litiasis radioopaca),
urografa i ntravenosa (confirma una posible anomala funcional y til
en litiasis radiotransparente), cistografa, estudio metablico (til en
prevencin de recidivas), Te abdominal, ecografa transrectal, biopsia
prosttica dirigida, pielografa retrgrada, nefrostografa, cistoscopia,
flujometra, cistomanometra y citologa urinaria (MIR 07-08, 105).
9.4. Tratami ento
Es necesario restablecer el flujo urinario. La mayor parte de las veces
se real i za mediante l i totricia o correccin quirrgica. Si la obstruccin
es aguda y/o bilateral, la desobstruccin es urgente, y puede lograrse
mediante un sondaje vesical, talla vesical, catter ureteral o nefrosto
ma. En caso contrario, hay que valorar el grado de sufrimiento renal
urologia
y su reversibil idad. En los casos en que hay destruccin irreversible de
la va urinaria, es necesario realizar una derivacin urinaria definitiva
(Figura 28).
MANEO DE L UROPATIA OBSTRUCTIVA
INFRAVESICAl
(globo vesical)
+
SONDA VESICAL
O ClSIA
No dilatacin
Estudio mdico
+
SUPRAVESICAl
(no globo vesical)

Dilatacin unilateral
A
Dilatacin
bilateral
Birreno Monorreno
j
Estudio (UIV, T)
Fracaso renal
Derivacin de va
urinaria superior
(nefrostomfa/doble J)
Figura 28 Proceim;ento de actuacin frente a la uropatla obstructiva
39
Orl(ntaclon
MIR
Tema de reciente
introduccin, sencillo
y muy rentable. Se debe
incidir en los factores de
riesgo, el tratamiento
y sus contraindicaciones.
Es conveniente conocer muy
bien las preguntas aparecidas
con anterioridad.
7 Preguntas
- MIR 06-07, 97
- MIR 05-06, 108
- MIR 04-05, 108
40
1 0.
DISFUNCi N ERCTI L
(
o
l
la causa ms frecuente de disfuncin ercti l es vascular.
la enfermedad endocrina ms relacionada con ella es la diabetes mellitus.
El sildenafilo est contraindicado en pacientes que toman nitratos o frmacos donadores de xido ntrico, en
pacientes con infarto agudo de miocardio (lAM) en los ltimos seis meses, y en pacientes con insuficiencia
cardaca grave o angina inestable.
1 0. 1 . I ntroducci n
La disfuncin erctil (DE) se define como l a incapacidad persistente o recurrente para conseguir o mantener l a sufi
ciente rigidez del pene que permita una relacin sexual satisfactoria. Debe tener una duracin mnima de tres meses.
1 0. 2. Preval enci a
En Estados Unidos, en un estudio en varones de entre 40 y 70 aos, se estim que l a prevalencia global era del
52%. La prevalencia en Espaa se estima en 1 . 500.000 a 2.000.000 varones, en torno al 1 2, 1 %.
1 0. 3. Eti ol oga
Se puede clasificar en (MI R 04-05, 1 08):
Orgnica. Causas vasculares (las ms frecuentes 60-80%), neurolgicas ( 1 0-20/), hormonales (5-1 0%) o
locales.
Psicgena.
Mixta. En la mayora de los casos de etiologa orgnica se aade un componente psicolgico.
1 0.4. Factores de ri esgo
Edad: factor independiente.
Diabetes: es la enfermedad endocrina ms frecuente asociada a disfuncin ercti l . Significa una probabil idad tres
veces superior de presentar DE. Estn impl icados mecanismos vasculares, neuropticos y disfuncin gonadal.
Enfermedad cardiovascular: cardiopata, hipertensin arterial, enfermedad vascular perifrica y descenso del
colesterol HDL se han relacionado de manera clara con la disfuncin erctil.
Tabaquismo: factor independiente.
Secundaria a frmacos: aquellos que causan hi perprolactinemia, que di sminuyen los niveles de testosterona,
psicotropos y antihipertensivos.
Secundaria a consumo de drogas: cocana, herona, etc.
Trastornos afectivos: depresin.
1 0. 5. Di agnstico
El diagnstico debe basarse en los siguientes componentes:
Historia clnica y sexual: i nvestigar los posibles factores de riesgo
impl icados.
Exploracin fsica: encaminada a descartar enfermedad vascu
lar, enfermedades neurolgicas, trastornos genitales y endocri
nopatas. En varones mayores de 50 aos se i ncl ui r tacto rectal.
Determinaciones analticas: glucemia basal, perfil l ipdico, testos
terona total y l ibre y prolactina. Adems, es conveniente solicitar
hemograma, funcin renal y heptica.
Pruebas especializadas: nicamente en ocasiones muy seleccionadas.
1 0.6. Tratami ento
Se puede estructurar en tres escalones o etapas, que sern superadas en
funcin de fracaso del escaln previo.
Frmacos orales
Citrato de si ldenafilo: se considera actualmente como el tratamien
to farmacolgico de eleccin en l a DE. Hoy en da existen nuevos
frmacos basados en l, como vardenafilo y tadalafilo. Se trata de
Pacientede 6J aos, en tratamiento a demandacon citatodesildenafilo por presen
tar disfncin erctil de aos de evolucin. Seale cul de los siguientes frmacos
NO asociara en ningn caso a su tratamiento:
1 ) Amiodarona.
2) Verapamilo.
urologia

un i nhi bidor de la fosfodiesterasa tipo 5 (PDE5). Induce la relaja
cin del msculo liso del cuerpo cavernoso, l iberando xido ntrico
(NO). El NO l i berado por el endotelio vascular y por las terminacio
nes nerviosas no adrenrgicaslno colinrgicas es el principal neu
rotransmisor de la ereccin. Precisa de deseo sexual y estimulacin
previa para su efecto.
Las contraindicaciones absolutas de si l denafilo son:
Administracin concomitante con nitratos o frmacos donadores
de xido ntrico por el riesgo de hi potensin grave (dinitrato/mo
nonitrato de isosorbide, molsidomina, nicorandi l , nitroglicerina,
nitroprusiato sdico) (MIR 06-07, 97; MIR 05-06, 108).
Pacientes en los que est desaconsejada la actividad sexual (an
gina inestable, insuficiencia cardaca o i nfarto reciente, hace
menos de seis meses).
Apomorfina: agonista dopami nrgico que acta a nivel central so
bre el mecanismo de la ereccin. Est contraindicado en sujetos que
tengan desaconsejada la actividad sexual .
Segunda l nea
Terapia intracavernosa: al prostadil (PGE1), mediante inyeccin di
recta en 105 cuerpos cavernosos. Otros frmacos son l a papaverina
y la fentolamina.
Tercera lnea
Ciruga de revascularizacin (venosa, arterial). Implante de prtesis
de pene.
J) Digoxina.
4) lndapamida.
5) Mononitrato de isosorbide.
MJR 05-06, 108; RC: 5
41

Вам также может понравиться